Vince's Best Tips

Sign up to receive my free 7-part "GRE Prep For High Scorers" email series.

GRE For High Scorers, Part 6: GRE Issue Essays

Vince's 2 main options to help you with your GRE prep are  HERE .

Vince's GRE Issue Essay Template

VIDEO: Watch me brainstorm a real issue essay prompt for an example of the Issue Essay template I describe in the below article.

In this article, I'll talk about my favorite template for writing a high-scoring Issue Essay. These tips will work best for you if you're a good writer; however, anyone can benefit from them.

First, here are two to-dos for you:

Action item #1: Read all sample essays and commentary in The Official Guide , as well as those in the  Verbal Practice book. These are great models for your writing since you can see what the ETS graders reward. Pay very close attention to the grader commentary.

Action item #2 :  Read and brainstorm two official  Issue Essay  prompts every time you study.

Bonus: learn more new words with my GRE vocabulary flashcards app !

Ok, assuming you've gotten started on the above, I want to give you some insight about what you need to do to get a 6.

you planning your essay?

GRE Issue Essay Example That Received A Perfect Score

You may notice that many Issue Essay prompts make statements that are difficult to fully support. The example and response I want to use first is this one about technology (link is to the full sample essay on the GRE's website). Go ahead and read the prompt, then read the "6" response just below it. Now, let's think about its statement:

As people rely more and more on technology to solve problems, the ability of humans to think for themselves will surely deteriorate.

Notice that this statement - like many Issue statements - would be difficult to agree with 100% of the time. Good responses will acknowledge the complexity of the issue and respond in an insightful way to that complexity. This response does that partly by discussing the reasoning that might be used by someone who agrees with the statement (in paragraph 2):

The statement attempts to bridge these dramatic changes to a reduction in the ability for humans to think for themselves. The assumption is that an increased reliance on technology negates the need for people to think creatively to solve previous quandaries. Looking back at the introduction, one could argue that without a car, computer, or mobile phone, the hypothetical worker would need to find alternate methods of transport, information processing and communication. Technology short circuits this thinking by making the problems obsolete.

Good writers often begin this way - they talk about their opponents' views, then respond with their own . This provides context and a framework for their argument. Writing a persuasive essay without addressing the reasoning of other points-of-view is like pretending your point-of-view exists in a vacuum with no one to challenge it.

essay frustration?

Now let's look at a paragraph that continues to acknowledge the complexity of the issue in an insightful way (paragraph 3):

However, this reliance on technology does not necessarily preclude the creativity that marks the human species. The prior examples reveal that technology allows for convenience. The car, computer and phone all release additional time for people to live more efficiently. This efficiency does not preclude the need for humans to think for themselves. In fact, technology frees humanity to not only tackle new problems, but may itself create new issues that did not exist without technology. For example, the proliferation of automobiles has introduced a need for fuel conservation on a global scale. With increasing energy demands from emerging markets, global warming becomes a concern inconceivable to the horse-and-buggy generation. Likewise dependence on oil has created nation-states that are not dependent on taxation, allowing ruling parties to oppress minority groups such as women. Solutions to these complex problems require the unfettered imaginations of maverick scientists and politicians.

I like this paragraph a lot because not only does the author make an insightful point that technology gives people more time to think, but that technology itself creates problems that require thinking. This is a step up from what I think the average writer might do - just cite an example of technology that helps us think or that we need to think to use. Now, you don't always need this level of insight, but it's a good example of what might separate a 6 from a 5. Compelling reasoning and depth of thought are rewarded.

I also like that this essay creates a critical context with the first paragraph, defining the scope of what it's going to discuss. It's the one I most talk about if a higher-scoring student wants some kind of GRE issue essay template, too. It's nice to have a go-to structure when you've only got 29 minutes and 37 seconds and the clock is relentlessly ticking...

Another example I like in the Verbal Reasoning Practice Book is the 6 essay response to a prompt about "People should obey just laws and disobey unjust laws". The author spends some time questioning the context to apply "just" - i.e., what is more important, being just to the individual or just to society? The author also questions how to define "just": if a society is brainwashed, can "just" have any meaning? I like this approach, since, again, it acknowledges the complexity and complications of forming a position on that issue.

essay writer with pencil

How To Effectively Choose And Use Examples In The Issue Essay

Many of my students find it difficult at first to think of relevant examples to support the arguments they make when writing the GRE issue essay. In this section, I’ll give you one of my favorite GRE essay tips: how to choose strong examples. Again, just so we have some context, here’s a sample Issue essay prompt: “Employees at all levels of a corporation should be involved in that corporation’s short and long term goal planning.” Now, one trap I want you to avoid falling into is thinking you have to have specialized knowledge of the topic. You might say to yourself that you’ve never worked for a corporation and that you don’t know how corporations typically plan. The good news is that you can still write a good essay about the topic using what you do know.

For example, let’s say you mostly agree with the statement and wanted to think of an example you could use. You could use an example from real life – perhaps you could talk about how employees at Google are encouraged to use 20% of their time to work on projects they think will benefit the company. Even if you couldn’t think of a real life example, you might use a hypothetical example and talk about how if an airline asked all employees about its plans for the future, flight attendants might provide unique insights into what customers like and don’t like . One strategy I like to recommend is to instead of thinking of what the perfect GRE issue essay examples might be, think about what you know well and see if it could fit. This will encourage you to choose examples you know well, making it easier to write insightfully about them. It might be a good idea to make a list of your personal “go-to” topics – things you can easily talk or write about. You’ll find that you can use many of them for many essays – don’t let the first thing that comes to mind box you in. Essay topics are designed so that almost anyone can write about them. For example, I love reading  The Economist . Since I read it every week, I usually have been thinking about some current events that I can apply to many Issue topics. Also, reading like this builds up a storehouse of information that you can dip into, making it increasingly likely you'll be inspired when a random topic pops up on the screen.

researching essay examples

How Long Should GRE Essays Be?

In general, the more you write, the better. 500-600 words is a good goal for most people, but write more if you can!

Good writers tend to illustrate and explain well, which equals more words. If your essay paragraphs seem short, try explain why more and writing more detailed "for example" sentences. Your reader will find your argument more compelling, and the GRE computer algorithm* that contributes half of your essay score will reward you as well!

*Yeah - if you didn't know, both a robot and a human read and grade your essay, and your score will be the average of their scores. :)

How Do I Improve My Issue Essay Score?

Remember, the GRE issue essay examples you choose matter, but they must be used skillfully. Practice brainstorming lots of different essay prompts from the ETS website to get used to coming up with examples that you can use to support your point of view.

Improving your score generally will mean improving one of these five grading criteria:

  • How compelling is your argument?
  • The examples that support your point
  • Organization / transitions
  • Using standard written English

In my experience, the GRE's analytical writing assessment is the one part of the test that students tend to under-prepare for. No matter how you prepare for the essay, make sure that you at least write a few before you actually take the real GRE.

P.S. I can help you with your essays, but I will charge you for my time. If you don't want to work with me personally, you can ask for feedback on Reddit ( r/GRE ) - it's the most popular and active GRE forum I know of.

reddit logo

(Reddit is nerdy but also useful.)

Final Issue Essay Thoughts

Check out all the 6 responses in the ETS books (there are five "6" responses for the Issue task, and five for the Argument task). These will give you more ideas about what the highest-scoring essays do. Just keep in mind that these are paradigms of "6" responses... you don't always have to be that good to earn a "6". Again, I think one of the most valuable things you can do is to find an essay structure you like and create your own issue essay template out of it, so you have a "go-to" structure on test day.

If you enjoyed this free guide, check out Vince's other free GRE resources .

And, speaking of writing stuff, our graduate school admissions essay expert, Lauren Hammond, can help you write your personal statement !

Testimonial: "I wanted to improve my analytical writing score for the GRE and decided, with one week left before the test, that I should consult an expert for some last minute advice. Vince was awesome and provided me with so much feedback on the sample essays I sent to him. I ended up getting the score I needed, and I know meeting with Vince made that possible! Thanks again!" - Cayleen Harty

The Magoosh logo is the word Magoosh spelled with each letter o replaced with a check mark in a circle.

GRE Issue Essay: Strategies + 8 Real Student Essays with Scores

The content in this post applies in 2024 to the new, shorter GRE!

When you sit down at the computer on test day, the very first thing you’ll encounter is the GRE AWA Issue essay. For a lot of test-takers, this will feel daunting. But not you! In this article, Magoosh’s experts will guide you through the most important steps in attacking the analyze an issue task. In addition, we’ll take a look at student examples of the GRE “Analyze an Issue” task so that you can understand what gets a high score—and what doesn’t—on the official exam.

gre issue essay - magoosh

Table of Contents

An overview of the gre issue essay.

  • Top 5 AWA Issue Strategies

Student GRE Issue Essay Analysis: Prompts, Essays, and Grading Samples

So, what do you need to do for the GRE AWA Issue essay? Well, your goal is to read the prompt, then agree or disagree with the premise—and explain the extent to which you agree or disagree. Think you can’t prepare in advance? You’d be wrong! There are two main things you can do to get ready for the AWA portion of the GRE.

Review the Topic Pool

First, because the prompts are drawn from GRE’s published pool of Issue Essay topics , a bit of research will give you an idea of what to expect in terms of subject matter and presentation. Don’t try memorize all of them! There’s far too many. But! Do spend time browsing the topics and thinking about how to approach them.

Plan of Attack

Second, come up with a plan to navigate the GRE “analyze an issue” task. Not sure where to start? We can help! Here’s an example of a tried and true process for high-scoring essays that you can use to address any Issue task:

  • Read the directions carefully
  • Brainstorm and outline pros and cons
  • Choose a side
  • Select a concession point
  • Be sure to leave around two minutes for proofreading and editing

magoosh-lesson-video-icon

Top 5 GRE Analyze an Issue Task Strategies

Now that you have the basics down, let’s take a look at some more detailed strategies you can use to maximize your score on the GRE AWA Issue essay.

1. Be Organized

Even an impassioned, cogent response falls apart if it is not bundled into a proper essay format: An introduction, a few body paragraphs, and a conclusion.

2. Focus Your Paragraphs

The Introduction The Intro paragraph has a very limited purpose: The Intro should only introduce the topic and present a clearly defined thesis statement. The thesis will indicate your position on the issue. Your stance should be just ONE of the many points of view about the topic, not more than one. Often it is easiest for the writer—and the reader—if the last sentence in the Intro is the thesis.

The Body Paragraphs The 2-3 body paragraphs make up the bulk of analyzing the issue and should focus on using examples (ideally one per body paragraph) to develop and support your thesis. Make sure you use appropriate transitions and that your sentences link together cohesively so that by the end of each body paragraph you have persuasively—and clearly—shown how your examples supports your thesis.

The Conclusion The conclusion should be very short. In fact, it should only be a few sentences that recap your thesis and supporting points.

3. Keep It Engaging

Repetitive sentence structure makes for repetitive reading. Vary up the way you write—don’t be afraid to use a colon (or a dash), drop in a semi-colon, and vary up the syntax. A constant stream of noun followed by verb followed by adjective implies that you are a hesitant writer. You don’t want the overall impression your essay leaves on the graders to be a resounding meh .

4. Be Specific

Hypotheticals are fine, if you can use them to convincingly back up your point. However, that’s the tough part; “some people,” “mankind,” or “you” are dull and vague. Let’s say you are addressing this prompt: “Knowledge can sometimes be used for destructive ends.” Stating that “Oppenheimer’s knowledge of nuclear fusion allowed him to create the most destructive weapon the world had ever known” is far more impactful than, “scientists can sometimes use technology to hurt us.”

5. Stay On Topic

Perhaps the most important point (lest you wonder why you received a ‘1’ on your essay) is to keep your essay on topic. Imagine you had to respond to the mock prompt on knowledge I used above. If you begin talking about how technology is destructive because smartphones cause us to become insular… you have totally forgotten to answer the question, “Knowledge can sometimes be used for destructive ends.” Address the most compelling examples, yes—but the most compelling examples that relate directly to your topic!  

Now, it’s time to take a look at how sample essays meet (or fail to meet) the above criteria—and how this affected their scores. All of the following essays were written in response to the GRE Issue prompts , so check them out if you haven’t already, and then come back to analyze some examples!

Note: We’ve formatted the essays so that you can see the prompt and instructions first, then try writing your own response (this is great practice!). Once you’ve done that, click on the “essay and analysis” arrows to view examples of graded student essays and see how yours compare.

GRE Issue Essay Prompt 1: University Requirements

Prompt Universities should require students to take courses only within those fields they are interested in studying. Instructions Write a response in which you discuss your views on the policy and explain your reasoning for the position you take. In developing and supporting your position, you should consider the possible consequences of implementing the policy and explain how these consequences shape your position.

Student Essay

Some people believe that universities should put stringent policies in place that require students to take courses only within a chosen field of study, thus harshly limiting the breadth of knowledge that they are able to study. Concentrating on only one field is important in terms of developing expert knowledge and specialization, but it is also crucial that the student hone a well rounded knowledge of the nature of the world so that their field of specialization is accented with courses from outside disciplines as well. It is for this reason that I believe that students should focus their study on a specific field yet also be allowed and encouraged to accent and expand their specialized knowledge by sampling courses from other areas of specialty as well.

Our current globalizing world contains diversity of knowledge, culture and creed that is increasing at a rapid pace and in order to succeed in a world such as this, it is necessary to hone a diverse skill set of knowledge and expertise. Therefore, university policies should encourage students to accent their study of a specific discipline with outside courses that will enhance the breadth of their knowledge about the nature of the world. A student studying medicine, for example, clearly needs to focus the majority of their time on understanding the inner workings of the human body on a scientific level. However, it is also crucial for them to have a more general knowledge of the way in which humans function on an individual or cultural scale (i.e. psychology and anthropology), because effective doctors are not simply capable of diagnosing diseases, but can also interact effectively, with individual and cultural sensitivies, with their patients in order to provide the most well-rounded care. A mathematician who knows only about math and knows nothing about the ancient civilizations whose cultures discovered geography will be ill-suited to make math interesting to his future students or to understand the real world implications of the equations he slaves over daily. A one-dimensional course of study will only serve to foster bias and an uncritical approach to life in such students. Thus, because we live in a world that is multi-faceted, it is important for every specialist to learn a bit about specialities outside of their main discipline in order to augment their understanding of the world at large.

When universities provide a structure of encouragement for their students to augment their specified studies by selecting some courses from outside their discipline, there are some possible consequences, such as the potential for students to change their mind about what they want to focus on. Some may say this is an inefficient use of time and that it will confuse students. However, I would argue that it will foster a wider breadth of knowledge that is ultimately beneficial for any student; a student that started studying biology but then switched to psychology, for example, will always appreciate and pay heed to the importance of our life sciences and will not neglect to consider how the functions of the body may affect someone’s mental health. The existence of knowledge in a wider range of disciplines will only provide the student with more information with which to take charge in a world that is highly complex and rapidly changing all the time, and so allowing them to experiment a little and change their mind once or twice is to their benefit rather than to their detriment.

In conclusion, I disagree that universities should require students to take courses only within their specific, chosen field of study. When students are able to focus their study on one specific topic but then augment it by sampling courses from other disciplines, their knowledge becomes more wide ranging and interdisciplinary, thus providing a better foundation for them to succeed in a rapidly globalizing world. While they may change their minds as to their preferred topic of study one or two times, they will ultimately succeed by having a wide breadth of knowledge that will teach them to approach the world without a subject specific bias. Overall, it is best that universities allow their students to take courses outside of their chosen course of study in order to diversify their pallate of knowledge.

Issue Essay Analysis

This GRE Issue essay starts off with a strong intro that clearly articulates the author’s position. The essay is also very long, and the body paragraphs well developed. In terms of ideas this is a strong—though if slightly limited—essay. It makes a compelling case for interdisciplinary learning. A physician studying anthropology will be more culturally sensitive; a psychologist who studied biology will have a great appreciation for the biological underpinnings of the psyche. The writer justifies this well-roundedness in terms of relevancy: a one-dimensional person will struggle in our complex, globalized world. As well thought out and supported as these points, they are far too similar, and this essay would have benefited from picking another example that argues in favor of allowing students to take courses outside of their majors. Another flaw is the essay doesn’t directly addresses the directions: “should consider the possible consequences of implementing the policy.” Is a world of well-rounded, complex individuals the consequence of allowing students to choose subjects outside of their majors?

Stylistically this essay is not perfect, and I have some minor grumblings.

Improve your GRE score with Magoosh.

The ongoing debate about whether a university should require students to take courses only within their fields of study or take extra classes to fulfill graduation requirements is an interesting one. There any many valid arguments to each side and it is not a simple black or white choice when deciding who is right. However, by requiring students to only take courses within their major, it allows for students focus on taking classes that are only applicable to their future careers and allows them to save money in a time where saving money is equally important to a college degree.

In many situations, students will finish high school and go on to college with an idea of what they want to do with their life. For students who are in majors such as engineering or the a science field such as chemistry or biology, it is important to for them to stay on top of all of their course work because of the higher number of courses that they must take in order to fulfill the university requirements for a degree. Many of these students knew before they entered college that this would be the case and gladly accepted that challenge, however by requiring students to take extra general education classes to fulfill their diploma requirements seems counter intuitive to a level of education where students are beginning to focus and narrow in on their future career goals. By forcing say a engineering student to take music theory or British literature just simply to fulfill a general education requirement and having that class conflict with a engineering major course seems to prevent these students from coming to college and fully obtaining their goal as quickly as possible.

The other aspect to consider is the financial aspect. In many of these situations, the students are under pressure to finish their degree as soon as possible because of many state budget cuts to education which limit the number of classes offered with in their major. Not only does this mean extra classes that students must take and thus more money they have to spend because tuition is usually based on a per unit fee, forcing these extra classes upon can have a longer impact if they are forced to stay longer in college than they originally assumed they would. College already charges an extremely large amount to attend and that already does not take into account the other expenses that students have to pay (such as room and board, food, and books), but adding on extra semester, quarters, or even years because a student had to take general education classes instead of strictly major classes is an unfair system to put a student through.

As with any situation though, there are always exceptions to the rule. For one not every student enters college with the same career focus and direction as their peers. Many students will come into college unsure of the direction they want to take and many students who think they know what direction they want to go, end up changing their minds (sometimes multiple times). By requiring students to take classes from a broad range of spectrums, Universities can help students narrow down what career path they may want to follow. Many times students may have a preconceived notion of what a subject may be about and not want to try it, yet by requiring it, they may be able to find themselves in a new class with something they may choose to pursue in the future, something they perhaps never would have considered. There is also something to be said about being able to take higher education classes simply for the benefit of wanting to learn about something that interests you. College allows you to do that and by making it a requirement, it allows students a bigger chance to do that.

Overall though, universities that force students to take upwards of 10-12 general education classes just to fulfill a requirement for their diploma seems unfair. When a student comes into college with a specific end game in site, the universities should not hinder their goals by overloading them with extra requirements and instead focus on helping hem obtain their goals as quickly as possible. The time and financial benefits that could be reaped by not requiring students to take these classes could have a direct impact on the success of all students as well as the future communities they intend to help.

Score: 5.0 This essay covers most of the bases: it offers analysis on both sides of the issue, it throws in a few sentences that address the specific instructions, and it, for the most part, clearly articulates a position. The essay does not wow with thorough analysis, great sentence variety (or indeed any stylistic flourishes). In other words, it gets the job done without making too many missteps.

While I award this essay a ‘5’, there are moments when that score seems shaky. This is not mainly due to the ideas (though the generalizations don’t help: “As with any situation though, there are always exceptions to the rule”); at times the sentences become overloaded and tend to digress.

Word choice could have also been a little more dynamic. “Large”, “bigger”, etc. could be spiced up a little more: “astronomical”, “excessive”, etc.

In addition to making the sentence more readable, and varying up the syntax a little, the essay could have been improved with a little more analysis. I would have like to say more than taking more courses is expensive. Sure, that is a totally valid point, but to spend an entire paragraph on it the overly long first paragraph about students who are not engineers as well.

Additionally, the last body paragraph is confusing: “There is also something to be said about being able to take higher education classes simply for the benefit of wanting to learn about something that interests you. College allows you to do that and by making it a requirement,it allows students a bigger chance to do that.” Is the author implying that colleges shouldn’t require students to take only course in their field (which would go against the main point of the essay)? And by saying that colleges make “it a requirement” that college require students to take courses outside their field?

Had this paragraph been a little clearer and had the writer expanded the scope of the financial issue, this essay—along with a little more dynamic writing and sentence variety—could get at least a definitive ‘5’, if not a ‘5.5’.

Liberal arts colleges and professional schools often debate whether they are required to develop well-rounded individuals. The primary purpose of universities is to establish the ground work for future field experts and specialists, meaning the developing into other fields would detract from the development of specialization. A basic understanding of how to delve into other fields is all that’s necessary.

A college degree in a field suggests that a graduate has the basic understanding of a specialized field, and they may continue to develop into a true expert. At every level of the collegiate process, students have further expansion into their speciality. For instance, science majors start with basic fundamentals that are required for latter learning. They soon go off into their own fields, isolated from the humanities and, often, other science majors. Because students usually have only four years to achieve a set requirement of tested standards in a particular field, universities must push students into their fields quickly. There simply isn’t enough time to truly explore all the possible fields of study at the university level. Exploratory learning shouldn’t be required as it doesn’t serve any purpose when the student won’t continue to explore in those extracurricular fields.

If a student were to only hole themselves away into the fields of physics, they may never truly understand how their physical knowledge relates to society and the social world. Universities tend to have to weigh this “roundedness” against the need to produce future field experts. The outcome is introductory classes that relate to your field, but intertwine with other fields of study, and push students to explore on their own time. These initial exploratory classes would be necessary for any field of study anyway, as creativity and individual pursuit is essential for any expert to further their field’s knowledge.

These exploratory classes are necessary for students to apply their growing expertise, but leaving their fields of study should be done on their own because they can only expand into the elementary levels of other fields within their time restraints at the university level. In this way, students aren’t led by the hand through fields they aren’t interested in, but they would still have the capability to explore their fields if they truly were intrigued. Allowing students to create their own directions, intertwining their interests, creates dynamic individuals who are happier with their degrees and more productive to the world through their specialization.

Universities are meant to develop future experts and specialists in particular fields of study. They should lay the groundwork for students to be able to explore of fields, but not in a way that detracts from their field’s work. At a moment when their time is so precious, students can’t afford to be left behind in their fields as they are forced by curriculum to explore unwanted alternatives.

There are some things about the essay that I like: it brings up interesting ideas relating to the prompt. Do specialists with “roundedness”contribute more to their fields than those specialists who focus only on their fields? The sentence variety makes things flow along nicely, until the middle of the essay, where the author becomes vague. Indeed, at times I’m not sure which side of the prompt the author is arguing.

For example, at the end of the second paragraph he states: “Exploratory learning shouldn’t be required as it doesn’t serve any purpose when the student won’t continue to explore in those extracurricular fields.”

The very next sentence—the first sentence of the third paragraph—says the exact opposite: “If a student were to only hole themselves away into the fields of physics, they may never truly understand how their physical knowledge relates to society and the social world.” Suddenly,the paragraph is arguing against what the previous paragraph stated.

The second to last paragraph is weighed down in abstractions, without a useful specific example to clear things up. Consider the topic sentence: “These exploratory classes are necessary for students to apply their growing expertise, but leaving their fields of study should be done on their own because they can only expand into the elementary levels of other fields within their time restraints at the university level.” There is a lot going on here, and I really had to reread the sentence several times to get what the author was saying. The ETS graders won’t take this much time. And given that the essay has already pulled an about-face in the previous paragraphs, makes this sentence even more obfuscatory.

The conclusion is much clearer than the rest of the essays, and allows me to understand what the essay was trying to say alone.Compare the clarity of this sentence to the one I mentioned in the previous paragraph: “They should lay the ground work for students to be able to explore of fields, but not in a way that detracts from their field’s work.”

So how to grade an essay like this? Strong analytical skills, sophisticated writing, and solid organization….yet, a contradictory—and at times muddled (the clause in the intro, “….meaning the developing into other fields) leads to a confusing essay.

The author states that students should only take classes within their realm of study. Although, students may gain more of a grasp on what they are studying, this requirement fails to take in what students can learn outside of their required classes. To say that students can only take classes within their concentration is occluding them to knowledge that they may learn in other fields of study.

For example, universities typically require students to pick their major, as well as a minor. Some programs may also require students to select a few elective classes as well, so students can establish themselves as more rounded individuals.

Also, taking classes outside of a student’s field of study may help boost the student’s overall GPA. For example, if a student has an in major GPA of 2.5 and an out of major GPA of 3.2, then the overall GPA will increase. However, it could be vice versa as well. If someone isn’t doing that great in their elective classes, it could bring their overall GPA down.

If this policy is implemented, the consequences may be severe. One consequence could be that a student may not be able to graduate on time because they may not have enough credits. Or they may not meet the GPA requirements to graduate because they failed a few classes within their major.

If the university decides that students can only take courses within his or her chosen field of study, then the university may not produce well rounded individuals.

This essay is an example of a 4.0—just barely—that is undeveloped and thus on the short side. It is not an example of a longer, totally one-sided ‘4’ that ignores the directions (notice how the final body paragraph addresses the “consequences” mentioned in the instructions).

What the author has written is an intelligent response to the prompt. She doesn’t simply agree with the prompt, but takes the opposing side, providing support (“To say that students can only take classes within their concentration is occluding them to knowledge that they may learning other fields of study.”). In passing, I should mention that “occlude” is used incorrectly. This is not a major problem, but remember that, if you use GRE words, make sure you know how to use them correctly.

I do not agree with the stated policy to allow students to only take course within their chosen fields of study. Instead I feel that students should should have the opportunity to take course outside of their major for the following reasons.

First, I feel that taken course outside ones major gives students variety, and exposure to experiences or interactions they may not have considered previously. Take for example Lisa, an engineering student who spends countless hours studying. Realizing that she needed a change of place an outlet of some sorts decides to take a modern dance course just for fun. What ultimatly was that Lisa learned to relax which interned helped her study more effectively and perform better in her engineering course.

Then take Monique, a political science major who doesn’t know how to swim. decided to take a swimming course and not only learned to over come her fear, but gained confidence in other other aspects of o her live.

Thirdly, lets consider Jason, a physics major who only took courses in his major. He became such an expert in his field us study, but became increasing socially award because of his inability to converse or relate to his peers.

In the even both Lisa and Monique were not able to take course outside of their major, I fear that they would have succumb to the pressure that sometimes too often over takes students adjusting to university lift. By deviating from their mandatory set of course they found a renew focus and inner strength that they may have never know before. Jason however, didn’t fair as well due to his strict focus in University

University is about diversity and gaining new experience for growth and development. Not being allowed to explore this diversity limits the over experience and potential stunts the education growth and perspective of students

Grammatical errors and spelling mistakes mar the effectiveness of this essay. Specifically, commas are misused (or not used at all), incorrect words are used (“interned”, “award” vs. “awkward”). I think many of these mistakes can be remedied if the student spends some time editing.The point in editing isn’t to catch the nitpicky errors but the glaring ones (of which this essay has many).

Next, the essay has very predictable development: take one-side of the prompt, and then come up with three hypothetical examples to support the point. There is zero analysis. This essay could have been improved and gotten within striking range of a ‘4’, or at least a ‘3.5’, had it simply addressed the instructions: “consider the possible consequences of implementing….” Of course, addressing the grammatical and spelling errors would have helped the essay.

Prompt 2: Lasting Legacy

Prompt Those who see their ideas through, regardless of doubts or criticism others may express, are the ones who tend to leave a lasting legacy. Instructions Write a response in which you discuss the extent to which you agree or disagree with the statement and explain your reasoning for the position you take. In developing and supporting your position, you should consider ways in which the statement might or might not hold true and explain how these considerations shape your position.

A famous author once remarked that “Winners never quit and quitters never win”. People who see their ideas through, however unpragmatic it may be considered by others are the ones who have truly made a difference.

History is replete with examples of people who were perceived as crazy, illogical and even insane by laymen, yet when their ideas were sedulously worked upon, by the creator , day after day, combined with long hours of toil, the result was nothing, short of marvelous.

Lets’s take the example of the Indian freedom struggle fought by Gandhiji on the basis of Satyagraha. It was very difficult for the Britishers to assume that India would be freed one day under the leadership of a loin cloth covered ordinary looking man without the use of weapons or bloodshed. The reason that Indian freedom could be achieved was the unflagging determination of Gandhiji and the uncommon methodology used of winning freedom by peace and not bloodshed.

Looking not far, I can recall the example of Galileo who was reviled and persecuted by the Church authorities for challenging the existing norms that pervaded the society that time. Galileo’s fierce determination , not to give up on his ideas even during harsh criticism paved the way for modern space research.

Another convincing example is of the Wright Brothers. Who would have ever imagined that it is indeed possible to fly like a bird and traverse different parts of the globe. I am sure that the Wright brothers were reviled when they first came up with this idea of developing an aeroplane. But, again today their invention has become a legacy.

Though there are several examples of people winning through odds because of their determination and unflagging spirit and creating noteworthy inventions, there could be times when this may be the cause of much trouble. Consider the doggedness of Hitler.though he was criticised for his heinous atrocitities on the Jews, he still did not stop the atrocities. These are few examples when people with strong determination can create an ill legacy instead of a legacy.

The writing in this essay has a lot of punch and makes reading it easy. However, there is little to no analysis. Like many essays on this prompt, the essay takes an extreme position, and beyond a vague, jumbled mention of Hitler, does not address the instructions: “…you should consider ways in which the statement might or might not hold true.”

As an SAT essay goes—basically you can take a relatively strong position—this is a good essay. Even then, some of the examples lack persuasiveness: “I am sure the Wright brothers were reviled.” Maybe they weren’t (they actually were, somewhat), but to say “you think” vs. “many notable scientists mocked the Wright Brothers notion of human flight” makes the essay far more tentative than it should be.

Also, the examples are very sparse, especially Galileo. Some more development would have perhaps bumped this essay to a ‘4.5’. But without any analysis, and by failing to take into account the other side, this essay gets only a ‘4.’

Although, doubts and criticism expressed regarding a particular by others seem valid at the particular time of inception of time, if the person follows through his idea or well cherished dream, then he may become success in his endeavor and leave a lasting legacy. So, people who see their ideas through, regardless of doubts or criticism others may express, are the ones who tend to leave a lasting legacy.

New ideas takes time to be accepted by general public, and during the time from the inception till the acceptance, the person who invented or discovered that idea, may be criticized or oppressed. Galileo was put into house arrest for his entire life for his heliocentric model of the solar system, because it came in direct conflict with the church’s geocentric model which regarded Galileo’s theory as heresy. Later, Galileo’s model was readily accepted. So, it’s really important that the people should see their ideas through criticism and doubts of others and shouldn’t be daunted, since other people are not connected to the idea or dream or feel the strength of idea in the same way as the person who invented that idea.

If a person doesn’t

This essay struggles from a lack of clarity. The first two sentences are overloaded with words, and so it is difficult for a reader to figure out what the writer is trying to say. Since the essay graders do not have time to figure out what you are trying to say, you will be penalized. Luckily, the thesis is clear—though it is an almost exact rewording of the prompt.

The Galileo example—while expressed in language that is clearer than that found in the intro—isn’t that developed. We learn that he was arrested and confined for heresy. The essay automatically assumes that this is the same as criticism. I would say the church’s actions against Galileo are a little stronger than mere criticism.

What saves this essay from a sub-3.0 is the final sentence, which discriminates between the person with the idea and those who only have an inkling of that idea. However, this idea is not explored in more depth (and doesn’t really connect to the Galileo example). Indeed the essay ends there.

Prompt 3: Risky Action

Prompt People should undertake risky action only after they have carefully considered its consequences. Instructions Write a response in which you discuss the extent to which you agree or disagree with the recommendation and explain your reasoning for the position you take. In developing and supporting your position, describe specific circumstances in which adopting the recommendation would or would not be advantageous and explain how these examples shape your position.

People should undertake risky action only after they have carefully considered its consequences.

People should not let their fears prevent them from taking important risks in life. Taking risks is what allows us humans to achieve success, joy and ultimate fulfillment. However, prior to taking any risky action, it is essential that people should carefully consider the consequences.

For example, there are some risky actions that are life-threatening such as skydiving. Of course, before you can begin to skydive, you must learn the basics of this sport. Additionally, by also studying what can go wrong during a skydive, and learning how to react to that scenario, that person will have the knowledge and ability to stay calm and hopefully make better decisions that will allow them to get out of a bad situation rather than falling into a panic.

This also pertains to decisions about money and business. Everyday people are making decisions that are ‘make or break’. For those who really understand the consequences of their actions, they are able to make a wiser decision that may have less of an impact on them if the business or investment deal goes awry. However, but not educating oneself, the consequences of one’s action are likely to be more severe.

Sometimes, knowing the consequences of an action causes fear that will stops us from taking any risky actions. As a result we miss out on potential successes and most of all “joy”. Therefore, by understanding the consequences, one can eliminate feat, learn how to react in a smarter fashion and lead a much more enriching life than if they had never taken those risks at all.

This is a decent skeleton of an essay. But that’s the problem—it is only a skeleton and the ideas need a lot more fleshing out if this essay is to get at least a ‘5’. For instance, in the skydiving example, the writer barely scratches the surface. What are some things that a skydiver could possibly learn to help them make this risky endeavor less risky? How much less risky would they make sky diving? Is there a point where something is so risky that even if we take measures to prevent disaster from happening that something bad could still happen (skydiving in bad weather, or bungee jumping in a country that offers low prices—and also low quality equipment). In calculating risk, shouldn’t we also weigh the payoff. For the skydiving example, is the thrill worth the danger, even if one has taken the necessary precautions and learned proper technique.

A Final Word

Now that you’ve reviewed student samples from across the spectrum of GRE Issue task grades, you’ll have a better sense of what you need to do to get those high scores! More than anything, practice will help you get the score you want on test day. So take a look at the Issue pool and a few more essay examples, pull up a blank document, and get practicing! Best of luck on test day as you master the GRE Issue essay.

Chris Lele

Chris Lele is the Principal Curriculum Manager (and vocabulary wizard ) at Magoosh. Chris graduated from UCLA with a BA in Psychology and has 20 years of experience in the test prep industry. He’s been quoted as a subject expert in many publications, including US News , GMAC , and Business Because . In his time at Magoosh, Chris has taught countless students how to tackle the GRE , GMAT, SAT, ACT, MCAT (CARS), and LSAT exams with confidence. Some of his students have even gone on to get near-perfect scores. You can find Chris on YouTube , LinkedIn , Twitter and Facebook !

View all posts

More from Magoosh

Top GRE Scores - image by Magoosh

49 responses to “GRE Issue Essay: Strategies + 8 Real Student Essays with Scores”

Supatat Hovanotayan Avatar

Hello Magoosh team

First of all, thank you for your amazing tips about the issue essay.

But I still have a question about this task “if I write only two paragraphs, and mainly focus on only one side” Can I still get at least 4 points by doing this

Thank you very much for your kindness and time

Magoosh Expert

Hi Supatat,

By “two paragraphs”, do you mean two body paragraphs? You should aim for an introduction, 2 body paragraphs, and a conclusion. In other words, you should have 4 paragraphs. 🙂 As long as you use strong examples and make your point very clear, you should be able to get 4 points on the exam even without a third body paragraph.

Akshata Lolayekar Avatar

When giving examples whilst supporting our point, can we mention an borrowed idea or opinion and elaborate on it in our own words? Let’s say I mention an idea from Yuval Noah Harari and credit him? Will this be considered plagiarism in any way

Hi Akshata,

You can definitely mention an opinion as long as you state the original source. For example, you can say: “According to Yuval Noah Harari, […]” and that would be acceptable. 🙂

Bayenah Al-shami Avatar

Hello Firstly, thank you for this wonderful article. I have a question which is: How can I say a concession point without making any contradictions to previous paragraphs? I hope that my question is clear. Thanks

Hi there! Thank you, we’re glad you found it helpful. 🙂 I’d recommend reading over the example essays in this blog post to see how they handle the concession point. In addition, be sure to check out our blog article 12 Tips to Ace GRE Writing as well.

Joe Bouzide Avatar

I have a question regarding where to include the concession point in my essay. Does it receive its own paragraph within the body of the essay, or does each supporting idea have a concession point paired with it? And do you include the concession point in the intro and conclusion as well?

Thanks, Joe

Hi Joe! You can add a third body paragraph that discusses your concession if you have time, but you can also just make a quick concession point, say at the end of your second body paragraph. Just remember that the goal is to use the concession to prove your point. The most common mistake is to spend too much time on the concession, so it can be safer to do less than more. I would not recommend bringing up a concession in the introduction or conclusion. It’s possible, but it’s just too risky. Use your concession to say, “While it may seem that people are distracted by their cell phones, they are actually socializing while looking at their screens. Therefore, technology brings people together.” Something like that is a strong, quick concession, whereas if you spend a paragraph going on and on about how people never talk anymore, you run the risk of arguing for the other side! Hope that helps 🙂

Mursal Rabb Avatar

Hi, It is OK to write issue essay from first person perspective?

There is no specific prohibition of the first person and some people do well on the essay and use the first person. But I tend to recommend avoiding first person language, especially “I think” and “in my opinion.” Both of these phrases tend to be redundant because you usually can take these phrases out of the sentence and your sentence will still maintain its meaning and grammar. You can completely avoid the first person and your writing will likely end up with a more sophisticated tone.

If you do use the first person, I’d recommend that you use it once in the introduction paragraph for your thesis, and that is it.

I recommend taking a look at some of the sample essays written on some topics. These are released by ETS, the testmakers, and will give you an excellent idea of what a great, good, and poor essay will look like. You’ll notice that the essays rated 5 and 6 do not have first person language but the other, lower scored essays do.

http://www.ets.org/gre/revised_general/prepare/analytical_writing/issue/sample_responses

Avinash Avatar

I guess I am a lot of thoughts to put on, but facing trouble to make my writing more persuasive. Can you please suggest how i can make my writing more persuasive as to better reflect my thoughts.

In the AWA issue Essay, being persuasive is all about using evidence. Anytime you make a claim, think of the reasons people might doubt that claim. Address all of those most obvious doubts. Also think about any questions people might ask you to get a better idea about what you’re saying in your essay, and why you’re saying it. Always put forth a very complete set of supporting details and argumentative evidence.If you think you won’t have the time or space to complete your argument within the time and pace limits of AWA, then choose a different argument, or find a way to simplify your argument.

Meredith Avatar

One set of directions states to “discuss the extent to which you agree or disagree with the statement…” I’m confused by “extent.” Does this mean that ETS simply wants us to take a side either in agreement or disagreement and explain why? Or by “extent” do they mean that it is okay to strongly disagree, or to somewhat agree, etc.

Hi Meredith,

The second option is more accurate–another way to think about “extent” is “degree.” So not only do your agree or disagree, but what are the limitations of that opinion? I hope that helps! 🙂

Lid Avatar

Can you write in first person on either GRE essays?

There is no specific prohibition of first person and some people do well on the essay and use the first person. But I tend to recommend avoiding first person, especially “I think” and “in my opinion.” Both of these phrases tend to be redundant. You usually can take these phrases out of the sentence and your sentence will still maintain its meaning and grammar. So, you can completely avoid first person and writing in a more sophisticated tone.

If you do use first person, I’d recommend that you use it once in the introduction paragraph for your thesis. And that is it.

I recommend taking a look at some of the sample essays written on some topics . These are released by ETS, the testmakers, and will give you an excellent idea of what a great, good, and poor essay will look like. You’ll notice that a 5 and 6 do not have first person but the other lower scored essays do.

I hope that helps! 🙂

Alyssa Avatar

Hi Chris! I have a questions about the intro paragraph/thesis statement. Do you have to include the points you plan on discussing in your body paragraphs in your intro/thesis?

It’s not necessary to state your points verbatim in your intro — in fact, it will probably save you time not to do so 🙂

Davut Avatar

My exam is on 13th February and I have about 1 month from now on. I tried to focus on verbal and math section more until now and did not spend enough time on AW section of the GRE. Would you recommend writing one essay per day to gain acceleration on practicing ?

Any suggestion would be appreciated. Thanks.

I am so sorry this didn’t get answered quickly, but hopefully our advice can help! I’d suggest that you first take a look at these ETS topic pools:

List of AWA Issue Prompts List of AWA Argument Prompts

Familiarize yourself with these topics, and then write several practice essays of your own using these ETS topics as a way to familiarize yourself with the questions and expectations. If you are careful to answer the actual question posed by the AWA tasks and you prepare yourself by knowing what will be expected of you on that day, you won’t have any trouble getting a good score. 🙂

Laura Avatar

Oppenheimer used nuclear fission, not fusion. 🙂 The GRE grader do not care if your facts are correct, though.

Alex Avatar

Dear Chris,

Firstly, thanks for keeping up with the blog. It’s been a great help.

Secondly, I was wondering if there is any way to insert special characters on the Gre essay software during the exam – such as those required in ‘vis-a-vis’ or ‘blase’ or ‘cliche’. If not, should these phrases/words be avoided? I’m from India and keyboards here don’t have these characters on them by default.

Chris Lele

That is a good question. I have no idea of the keyboards here allow you to do so. Regardless, I don’t think ETS will hold that against you. Of course, there is a computer grader, but maybe it has been programmed not to dock. Still, I can’t image ETS being so picayune as to do you for not having the proper diacritic.

Hope that helps!

Cornelia Avatar

One thing that concerns me when writing my essays in the issue part is that a lot of the examples that come to my mind are not that well-known in the Anglosphere. I’m German, and I often think of something German scientists or politicians did or said, events that happened in Germany or things taught in German high school. The example essays that I compare my essays to usually score high by drawing on a wide range of examples that are well-known in the US. Stating my examples, that the examiner has possibly never heart of, either requires a longer explanation, for which I don’t have time, or googling on part of the examiner.

What would you suggest? In theory, the GRE should not be culturally biased. But I am afraid if I simply drop unknown German examples, the examiners might be confused.

Thank you for your advice,

PS: To know what I mean, I thought of some examples for you. Let’s say the issue is about privacy and I refer to the surge in users of the Posteo.de email client, a Berlin-based start-up whose unique selling point is that they protect their clients’ privacy as much as possible. Or in an essay about rebellion I could refer to the way the German authorities dealt with house occupiers in Dresden in contrast to those in Berlin after the fall of the Berlin wall – the occupiers in Dresden were given proper rent contracts while those in Berlin were forcefully evicted, causing violent clashes with the police. Or when writing about technology, I might want to cite the website dawanda.de where people sell self-crafted goods. I know that there exists a similar format in the US – etsy – but I am not that familiar with it and would not feel comfortable writing about it and would prefer the German example. This issue comes up for me with almost every essay I write at least once!

Holing Avatar

I am on the same boat and would love to see this question answered!

Hi Holing and Cornelia!

I know this is a late reply, but hopefully it can help others in your positions. 🙂

It is perfectly fine to use non-US examples for the GRE essays, but you want to make sure you give relevant context and information on the events so that the reader doesn’t have to guess whether or not your example really applies to the point you are trying to make. If you can do that, then any examples from your own country should be fine. 🙂

Karishma Avatar

Hi, I have read in most sites that practicing essays is the best way to go for AW. But writing a full length AW issue essay or argument essay takes 30 mins each for a time limited atmosphere. So my question is while practicing from the ets pool of topics, do we need to write full length essays for every topic or just structuring and brainstorming on the topic and writing mock essays 3-4 times will be enough?

Margarette Jung

Hi, Karishma

30 minutes for each essay can definitely be tough to fit into your schedule! Doing quick structuring/brainstorming is a good alternative when you don’t have a lot of time. However, especially as you near your exam date, make sure to sit down and do a few full-length essays (not all in a row, but maybe one every few days) just so you can feel comfortable with the experience. I hope that helps! 🙂

Best, Margarette

Thanks Margarette!!

Hashim Avatar

Hello people of Magoosh,

I have a question about writing a thesis for an issue task. I noticed that in the video lesson, the thesis contained a statement indicating choosing a side. However, there’s no mention of the main points covered in the body paragraphs. Is that a good practice? Don’t you think that a reader ought to know what to expect in the body paragraphs just from reading the thesis statement?

Referred thesis: “a college curriculum should be designed around the career a student will pursue upon graduation”

Kevin Rocci

Excellent question! In a typical, untimed essay you definitely would want to let the reader know what is coming. The intro and thesis should give the reader some idea of where the discussion is headed and what will be discussed. This is a common practice in American essay writing.

But with the GRE, our strategies are a little different. Since we have such a limited amount of time to write an essay, we recommend spending as little time as possible writing the introduction and conclusion. The bulk of your time should be spent crafting the body paragraphs. As such, we only recommend stating your opinion or stance on the topic and not worry about prefacing your examples and reasons.

This isn’t to say that you can’t do this. If you are a quick writer and have the time, then you can definitely indicate what the main points of your body paragraph will be. 🙂

Happy Studying!

Lara Avatar

I just started practicing the AWA and am following the 90-day study plan for beginners. I’m trying my best to follow the outlined time structure you suggested in the videos, but in my first two essays I’ve always run out of time and always seem to produce mediocre work. Would you recommend that I practice writing without a time limit for now? Or should I just keep working with the time limit and would I gradually improve with more practice?

Hi Lara, Happy to help!

First, I recommend to keep practicing. Writing the essays on the GRE is a particular type of skill that needs lots of practice. So keep your head down and keep at it.

Second, if you feel like you need extra practice, try writing an essay more often. Instead an essay a week, write two. This will give you more opportunities for improvement.

Third, I recommend that you keep timing yourself. It doesn’t help to be good at writing an essay in an hour. We need to be good at writing an essay in half an hour.

One thing that I have done with my students in the past is have them write only an introduction or only an introduction and body paragraph in a set amount of time. So give yourself a time limit of 8 minutes and see if you can complete an introduction and body paragraph. This allows you to practice writing under time constraints and you can take baby steps towards completing an essay in 30 minutes.

I hope that this helps! Best of luck in your studies! 🙂

Marcel Avatar

I just started reading the book you recommended: On Writing Well, by William Zinsser. Although I would love read all of it, I don’t have much time to spare. Could you suggest what chapters would most benefit us for the GRE AWA ?

Good question! I think the grammar-related passages are important. As are the chapters that relate to crafting sentences and creating paragraphs.

Asma Maladwala Avatar

Hi Chris, Do you know if there are any sites where I can find high scoring sample essays? I’ve been practicing but feel as though I’m in a void as I have no point of comparison. Getting feedback from family and friends is helpful, but I’d just feel so much better if I could compare my essays to actual GRE essays. I could only find one sample set on the ets website…

It seems that only gre.org offers example essays. Just google “example GRE essays” and it should be the second hit.

Besides that there aren’t too many others I can think of that are online. Writing higher scoring essays, ‘5.5-6’ for blog posts is something I plan to do soon though :).

Veronica Avatar

Hello! I would enormously appreciate if you can clarify me this. Which link are you referring to in the following sentence?:

” For practical advice on practicing: the link below provides access to hundreds of essay prompts by ETS”.

I cannot find it anywhere and it would be of invaluable help for me to have these essay prompts in order to practice.

Thank you very much!

No problem :).

Here is the link: http://www.ets.org/gre/revised_general/prepare/analytical_writing/issue/pool

Verónica Avatar

Thanks for your quick response!

J Avatar

So I just found out ETS has started employing their e-rater technology. Thoughts?

Thanks for reporting that! Well, I hope it is better than the GMATs, which apparently counts number of words, a couple of transition sentences, etc. I guess time will tell.

emma Avatar

whats e-rater technology, mentioned by J, Chris??

Muhammad Usama Khan Avatar

Sometime it seems that we cannot write enough in the issue task.

If we practice one essay per day, who will rectify this and will tell us how to improve our score in analytic. So that we can BUT ALL feel confident to write essay with positive tone.

Yes, that is true, and indeed I need to write another post on generating ideas.

As for somebody to give you feedback, find a trusted family member or friend. Of course, that person would not want to read everyone of your essays, but as long as you get feedback every once in awhile that will help :).

Bhavin Parikh

This sentence is dead-on, “If you think you did poorly on the essays, that knowledge could very well affect your performance on the rest of the test.”

I recently talked with a student who was consistently scoring in the 80th percentile on math and verbal in practice. But he wasn’t prepared for the writing section on test day and it affected his concentration throughout the rest of the exam. He scored in the 60th percentile. Doing well on writing can definitely set a positive tone for the rest of the exam.

Yes, I am happy to hear that student’s experience echo my thoughts. Really, “Doing well on writing can definitely set a positive tone for the rest of the exam” is perhaps the greatest GRE tip that nobody has ever heard of.

typeR Avatar

Did u mean non-native below?? “Two of the preeminent prose stylists of the English-language novel were both native-English speakers.”

Ha! Yes, I definitely did. Thanks for catching that :).

Leave a Reply Cancel reply

Your email address will not be published. Required fields are marked *

www.800score.com

  • What is the GMAT?
  • GMAT Scores and B-Schools
  • How the GMAT CAT Works
  • GMAT Pacing Strategies
  • GMAT Tips & Strategies
  • Reading Intro
  • 1. Passage Classification
  • 2. Analyzing Paragraphs
  • 3. Passage Mapping
  • 4. Find the Main Idea
  • 5. Determine Purpose
  • Detail of the Passage
  • Definition of a Term
  • Support for a Premise
  • Function of Passage Part
  • Cultural Hard Ex. 2
  • Society Passages
  • FREE GMAT PREP COURSE
  • Science Medium Essay 1
  • GMAT Reasoning Intro
  • Most Strongly Supported
  • Argument Structure Questions
  • Ad Hominem Flaws
  • Argumentative Flaws
  • Assumption Questions
  • Parallel Reasoning
  • Parallelism Questions
  • Causal Arguments
  • Statistical Reasoning
  • Weaken Reasoning
  • Sufficient/Necessary
  • Paradox Questions
  • Modify Questions
  • CR Chapter Review
  • Grammar Basics
  • Sentence Correction Tips
  • The Three-Step Method
  • A. Introduction
  • B. Subject / Verb Separation
  • C. Collective Nouns
  • D. Plural / Singular
  • E. Neither / Either
  • F. Or / Nor
  • G. Subject / Verb / Object
  • H. Quantity Words
  • I. Sample Questions
  • B. Adjectives and Adverbs
  • C. Adjectives and Adverbs with Sense Verbs
  • D. Misplaced Modifiers
  • E. Sample Questions
  • B. Lists of Verbs and Parallel Constructions
  • C. Lists of Adjectives or Adverbs
  • D. Comparisons
  • E. Correlative Pairs
  • F. Sample Questions
  • B. Subject vs. Object
  • C. Who vs. Whom
  • D. Singular and Plural Pronouns
  • E. Possessive Pronouns
  • F. Objects of to be verbs
  • G. Relative Pronouns
  • H. Sample Questions
  • B. Verb Tense
  • B. Parallelism
  • C. Comparative and Superlative Forms
  • D. Sample Questions
  • Sample Questions
  • Divisibility
  • Simplifying Rules
  • Exponent Expressions
  • Complex Expressions
  • Inequalities
  • Working With Formulas
  • Intersecting Lines
  • Coordinate Geometry
  • Permutations
  • Problem Variations
  • Combinations
  • Combination or Permutation
  • Simple Probability
  • Independent Events
  • Dependent Events
  • Working Backwards
  • Using Outcomes
  • Other Scenarios
  • Statistics Intro
  • Normal Distribution
  • Chapter Review
  • Introduction
  • Standard Deviation
  • Trick Questions
  • Practice Questions
  • Seven Techniques
  • Ratio and Proportion
  • Distance, Rate, & Time
  • Graphs and Data Interpretation
  • Mean, Median and Mode
  • Integrated Reasoning-Introduction
  • Multi-Source Reasoning
  • Table Analysis
  • Graphics Interpretation
  • Two-Part Analysis
  • AWA Introduction
  • Dissecting Arguments
  • Finding Errors
  • Using Strategy
  • International Students
  • Fill Sentences
  • Qualification
  • Start Strong
  • Active Voice
  • Self-Reference
  • Vague Writing
  • Voice Shifting
  • Colloquialisms
  • Using Hyphens Correctly
  • The Apostrophe
  • Additional Essays
  • 10 Most Common Errors
  • GMAT Classes
  • Mission / About Us

Home » Free GRE Prep Course » Free GRE Prep Course » Template

ESSAY STRUCTURE

As with the Issue essay, there is no single “correct” way to organize an Argument essay.

Video Courtesy of Kaplan GRE prep.

Introductory Paragraph (2-4 sentences)

Try to accomplish three goals in your introductory paragraph:

  • Briefly restate the argument in your own words.
  • Briefly trace the argument’s line of reasoning.
  • Indicate the extent to which the argument is logically convincing.
  • If possible, sum up your arguments in one sentence (or two brief sentences).

The following is a sample template for the first paragraph that accomplishes these goals:

The author concludes that____________, because ________. The author’s line of reasoning is that ______________. This argument is unconvincing for several reasons; it is____________ and it uses _____________.

First Body Paragraph (3-5 sentences)

In the first body paragraph, your goal is to critique one of the following:

  • The reasoning of the argument
  • One of the premises of the argument
  • One of the assumptions of the argument

The following is a sample template for this paragraph that accomplishes this goal:

First of all, ____________________________ is based upon the questionable assumption ________________________________. That _______________, however, _________________. Moreover, ________________________.

The purpose of the second paragraph is to address one of the following:

Secondly, the author assumes that_________________________. However, __________________________. It seems equally reasonable to assume that____________________.

Third (and optional Fourth) Body Paragraph

In this paragraph, your goal is to critique one of the following:

Finally, _______________________________________. The author fails to consider__________________________________. For example, __________________. Because the author’s argument _________________.

Final Paragraph (2-3 sentences)

In the final paragraph, your goals are to:

  • Summarize your critique of the argument
  • State the main point of your essay

The final paragraph is not the place to introduce new arguments or issues. Sample template:

In sum, I agree that______________________. However, ____________________; on balance, _____________________.

The numbers of sentences indicated for each paragraph are guidelines, not hard-and-fast rules.

Do not be a Parrot The transitional phrases used here are purposely simplistic; do not simply “parrot” them word-for-word in your essay or adopt a fill-in the blank approach. If you do, your essay might appear stilted or contrived.

Free COURSE

  • What is the GRE?
  • GRE Scores and Admissions
  • How the GRE CAT Works
  • GRE Tips & Strategies
  • Cultural Hard Ex. 1
  • 2. Society Passages
  • Science Medium Ex. 1
  • Science Medium Essay 2
  • GRE Reasoning Intro
  • Assumptions Questions
  • Critical Reasoning Review
  • Consecutive Numbers
  • Algebra Simplifying Rules
  • Statistics Introduction
  • Question Types
  • Quantitative Comparisons
  • Distance, Rate & Time
  • Analytical Writing Intro
  • Analysis of Issue: Content
  • Analysis of Issue: Structure
  • Analysis of Issue: Timing
  • Eliminating Fill Sentences
  • Analysis of Argument Questions
  • Analysis of Issue Questions
  • GRE VOCABULARY LIST
  • Text Completion
  • Sentence Equivalence
  • Sample GRE Tests

template for gre issue essay

244 5th Avenue #2638 New York, NY 10001 US Contact Us

GMAT Prep | GMAT Classes GRE Prep | GRE Classes LSAT Prep | LSAT Classes MCAT Classes SAT Classes

© 2023 | Terms

Testimonials

Free Resources

PrepScholar GRE Prep

Gre prep online guides and tips, gre issue essay: 4 steps to a perfect score.

template for gre issue essay

Of all the different kinds of questions on the GRE, the GRE Issue essay question can seem like the most daunting to answer completely correctly. Instead of choosing from a selection of already-made answers or filling in a numerical solution, you must write hundreds of words in an attempt to fulfill rubric criteria, knowing that there is no one right answer to the question.

To help make this Herculean task more manageable, we’ll go over the Issue essay GRE rubric in depth and offer our top GRE Issue essay tips to help you score highly every time.

Feature image credit: Bruce Berrien /Flickr

Do You Need a 6 GRE Writing Score?

In general, your GRE Writing score is the least important of your GRE scores. No grad school will require you to get a 6.0 on the essay. A 4.5 is a good GRE writing score for most schools and programs, regardless of the discipline. Even programs that have cutoff scores for writing-heavy programs, like UNC’s Media & Journalism graduate degrees , don’t require anything above a 4.5.

If you’re looking to emphasize your writing skills (for example, if you’re an international student whose first language is not English and you want to show that you can write well in English), a higher score (5.0+) can help. However, even in those instances, a perfect 6.0 score isn’t going to be necessary.

Some doctoral programs, like Harvard’s Education Ph.D. , might have higher average scores, but that’s a function of the students applying being strong writers (which you have to be to make it to the doctoral level), rather than the program itself requiring certain scores. For Ph.D. programs, you’ll have the opportunity to demonstrate your analytical thinking skills in other ways that are weighted more heavily than your GRE Analytical Writing score.

Learn more about what you’ll need to get into grad school with our article on grad school requirements !

Quick side note: we've created the world's leading online GRE prep program that adapts to you and your strengths and weaknesses. Not sure what to study? Confused by how to improve your score? We give you minute by minute guide.

You don't NEED a prep program to get a great GRE score. But we believe PrepScholar is the best GRE prep program available right now , especially if you find it hard to organize your study schedule and don't know what to study .

Click here to learn how you can improve your GRE score by 7 points, guaranteed .

Walt Stoneburner/Flickr.

What Goes Into a 6-Scoring GRE Issue Essay?

The best way to determine what is needed for a perfect Issue essay score is to take a look at the official rubric and go over how the human essay grader is rating your essay.

To show the differences between a passable Issue essay and a perfect Issue essay, I’ve created a side-by-side comparison of the criteria for a 4-scoring and 6-scoring Issue essay on the GRE.

To summarize the information above, a perfect 6 Issue essay:

  • Must make sense logically
  • Must be precise in its discussion of the issue and the author’s stance on the issue
  • Must include support for the author’s position that persuades the reader to the author’s point of view
  • Must be organized and flow smoothly from idea to idea
  • Must be well-written

In order to achieve a perfect score on the Issue essay, you must excel in every one of these areas.

body_excelatessay

Official GRE Issue Essay Example, Analyzed

Now we’ll take a look at a sample GRE Issue essay that’s already been assigned a score of 6 and find all the ways in which it fulfills the rubric. Doing this analysis will help show how the rubric is applied by taking the abstract criteria and showing concrete examples.

For the purposes of this analysis, we’ll be using excerpts from this officially-scored essay . Here’s the prompt the essay is addressing:

As people rely more and more on technology to solve problems, the ability of humans to think for themselves will surely deteriorate.

Discuss the extent to which you agree or disagree with the statement and explain your reasoning for the position you take. In developing and supporting your position, you should consider ways in which the statement might or might not hold true and explain how these considerations shape your position.

The sample essay we’ll be discussing argues against this statement, taking the position that rather than fearing technology will make human thinking obsolete, we should embrace the possibilities and human potential unlocked by technology.

I’ll next go over how each of the rubric criteria applies to this particular sample essay. The first item in the rubric is a holistic description of a perfect-scoring GRE Issue essay:

Rubric description : In addressing the specific task directions, a 6 response presents a cogent, well-articulated analysis of the issue and conveys meaning skillfully

This item is meant to be an umbrella under which the next four criteria can fall; if an essay meets each of the four non-general criteria listed in the rubric, then it will exemplify this holistic description. The above description is also useful as a catchall reminder of what a perfect-scoring essay should look like, since essay graders aren’t necessarily going through the rubric item-by-item for each essay.

body_essaygraderportrait

The first of the non-general rubric items has to do with how well an author makes her point of view clear throughout the essay.

Rubric description : A 6 essay articulates a clear and insightful position on the issue in accordance with the assigned task

The sample essay successfully meets this requirement in a couple of different ways. In the essay, the author’s position on the issue (a counterargument to the prompt) is articulated in a series of logical steps over the course of the entire essay as well as in a final thesis statement.

Starting with the acknowledgement that “technology has revolutionized the world” in the first paragraph, the author goes on to make the argument that “reliance on technology does not necessarily preclude the creativity that marks the human species” (paragraph three), demonstrating a firm grasp of the issue through a nuanced, rather than absolute, position.

With each succeeding paragraph, the author continues to develop her position on the issue with clarity and insight. The author expands the initial argument to claim that “technology frees the human imagination” (paragraph four) and “By increasing our reliance on technology, impossible goals can now be achieved” (paragraph five).

The author’s final statement on the issue condenses the author’s point of view into a single sentence: “There is no need to retreat to a Luddite attitude to new things, but rather embrace a hopeful posture to the possibilities that technology provides for new avenues of human imagination.” This last sentence is not only the logical conclusion to the author’s clearly stated position, but is in itself a clear statement of the author’s position.

body_technologyforthefuture

The next rubric item is concerned with how well an author develops and supports her points.

Rubric description : A 6 essay develops the position fully with compelling reasons and/or persuasive examples

As I noted in the discussion of the first rubric description, the author’s basic position in this essay is that we should not fear technology because it is new and unknown, but instead embrace it because of the possibilities it offers for our future. In addition to developing her position through an insightful position articulated through the essay, however, the author also does an excellent job of supporting her points with examples and reasoning. Here’s an excerpt from the essay that illustrates this development and support:

“The car, computer and phone all release additional time for people to live more efficiently. This efficiency does not preclude the need for humans to think for themselves. In fact, technology frees humanity to not only tackle new problems, but may itself create new issues that did not exist without technology. For example, the proliferation of automobiles has introduced a need for fuel conservation on a global scale. With increasing energy demands from emerging markets, global warming becomes a concern inconceivable to the horse-and-buggy generation.”

In the above excerpt, the author develops her point with three actions:

#1: She presents examples to support her point that efficiency is enabled by technology (car, computer, and phone).

#2: She explains what the existence of these examples implies (efficiency doesn’t mean lack of thinking).

Want to improve your GRE score by 7 points?  We have the industry's leading GRE prep program. Built by world-class instructors with 99th percentile GRE scores , the program learns your strengths and weaknesses through machine learning data science, then customizes your prep program to you so you get the most effective prep possible.

Try our 5-day full access trial for free:

#3: She follows up with further reasoning about new issues created by technology (technology means humans can tackle new problems, including new issues created by technology). This reasoning is then backed up by more examples (cars and increasing energy demand), starting the cycle over again.

The examples and reasoning the author employs in her essay are compelling not just because they are logically consistent with the author’s argument, but because they are explained in a way that makes this link clear. If the author has said “In fact, technology frees humanity to not only tackle new problems, but may itself create new issues that did not exist without technology. An example of this is the automobile and increasing energy demands because of it,” the examples would have still been present, but not explained in a compelling or persuasive way.

body_automobile

The third non-general rubric item drills down even deeper into the logic of the author’s writing and analysis.

Rubric description : A 6 essay sustains a well-focused, well-organized analysis, connecting ideas logically

While with the previous rubric item I discussed the necessity of having a logical connection between the author’s position and the support for that position, this item refers to the author’s skill in connecting different ideas throughout the essay.

In addition to having a logical progression of the analysis (which is captured under the first rubric item to some extent as well), a perfect-scoring Issue essay must also have logical transitions between ideas . A good example of this occurs in this essay in the transition between the end of paragraph two and the beginning of paragraph three:

“Technology short circuits this thinking by making the problems obsolete.

However, this reliance on technology does not necessarily preclude the creativity that marks the human species.”

The first sentence of paragraph three (“However…species”) connects the ideas of paragraphs two and three. The author forges a link between the two ideas by restating the last-discussed idea from paragraph two (technology does take away some problems) in a way that sets up the idea to be discussed in the next paragraph (reliance on technology doesn’t mean humans won’t think for themselves). Specifically, the author does this by using a transition word (“However”) to link a reference to previously discussed ideas (“this reliance on technology”) with a reference to what’s coming next (“technology does not necessarily preclude creativity”).

The tightness of the logical connection between the two paragraphs and ideas also means that the essay stays organized and focused on the task at hand (presenting the author’s position on and analysis of the issue).

body_dartfocused

The last rubric item assesses the writer’s overall skill in use of language and standard, error-free English.

Rubric description : A 6 essay conveys ideas fluently and precisely, using effective vocabulary and sentence variety. Demonstrates superior facility with the conventions of standard written English (i.e., grammar, usage and mechanics), but may have minor errors.

This final set of criteria has less to do with the topic being discussed and more on the writer’s ability to indicate precise meaning through appropriate use of language. The final sentence of the essay provides a good example of this:

“There is no need to retreat to a Luddite attitude to new things, but rather embrace a hopeful posture to the possibilities that technology provides for new avenues of human imagination.”

This sentence uses effective language (Luddite, avenues of human imagination) that precisely conveys meaning. For instance, “Luddite” is a term that is generally used to mean resistance to technology, but more specifically has its origins in a group of people who were worried about what advances in technology meant for human workers, so it is particularly appropriate for this essay about the effects of technology on human abilities.

Another reason I chose to use this excerpt is because while the author uses effective language, there are still some minor errors (as the rubric description allows for). In this sentence, “but rather” is used incorrectly because it refers back to the subject “There,” which makes no sense with the “but rather” phrase. One correct way to say this would be, “ We should not retreat into a Luddite attitude toward new things, but rather embrace a hopeful posture to the possibilities that technology provides for new avenues of human imagination.”

In a way, this rubric area is the “icing on a cake” domain—you can have a reasonably clear and insightful essay without a high level of skill in this domain, but if you don’t use language skillfully your cake of an essay is not going to taste as good and won’t score a perfect score. And if you try to load your essay with advanced vocabulary words without care for whether or not they make sense in context, you’ll end up with lumpy frosting that makes the cake worse than it would’ve been without the icing.

Don't let your words be the uneven frosting on the delicious banana cake of your essay! Tim Pierce/Flickr

4 Steps to a Perfect GRE Issue Essay

As a summing-up of all the information in this article, I’ll go over the four essential GRE Issue essay tips to reliably achieve a high score.

#1: Include a Clear Thesis

To fulfill the basic requirements of any GRE Issue essay task, you need to make your position on the issue clear . The easiest way to do this is with an introduction paragraph , or at the very least an introductory sentence at the beginning of your first paragraph, that outlines the issue and where you stand on it.

There is no explicit requirement on the rubric that you include an introduction and conclusion in your essay, and in fact ETS encourages students to be as freeform as fits the topic and task at hand. However, if you don’t start your essay with some kind of introduction and wrap up your points at the end with some kind of conclusion, you run the risk of being unclear about your position. Not only can this be a problem for the reader, but without a clear thesis statement at the beginning of your essay to keep you focused, you may find yourself meandering off topic, resulting in a disorganized and inconsistent essay.

Thus, we strongly recommend beginning your essay with at least an introductory sentence and wrapping it up with a conclusion statement. You don’t have to have entire paragraphs devoted to each, but it is useful to bracket your essay between an introduction and conclusion to keep your thesis front and center.

Keep your position as clear as possible in your essay. bazzadarambler/Flickr

#2: Preplan Opinions and Examples

ETS has published all prompts it will ever use for GRE Analytical Writing, which in the case of the Issue essay comes out to 152 unique topic/task combinations. Now, obviously it’s not feasible to write a practice essay for every one of the 152 possible Issue essays and memorize it in preparation for the test. On the other hand, it is very possible to prepare some examples and evidence ahead of time , as long as these examples and pieces of evidence are flexible enough to be useful for multiple different prompts.

Start by reading through the complete list of Issue essay prompts and noting any common themes. Some examples of topics that seem to come up again and again in GRE Issue prompts are the roles of government and public officials, the role of technology in our lives, and the role of education and teachers.

Practice forming opinions about subsets of these topics and thinking of evidence that can be used to support those opinions. You very likely already have opinions about some of these things already, like the role of technology in education, or the importance of government support for research. To prepare for the Issue essay on the GRE, however, it’s not enough to just have opinions – you need to be able to back up your claims and point of view with evidence or reasoning.

For instance, let’s say my pre-planned opinion is that humans relying on technology to solve problems has resulted in humans being able to think for themselves even better than before. In order for this position to be worth anything in a GRE essay, though, it needs to be backed up by reasoning or evidence.

For this particular case, then, I might preplan the evidence that the expansion in size and complexity of the human brain’s cerebral cortex occurred around the same time as humans began to use tools, which could suggest that as humans relied more on tools (technology), their brains actually had to become bigger and better at thinking than before. I could also choose to preplan reasoning to back up my point, like the fact that relying on technology to solve smaller problems pushes us to use our thinking to attack larger scale issues, whether philosophical or practical.

Even if prompts on the role of technology, or on other topics you’ve practice explaining support for, don’t show up when you take the test, you’ll be better equipped to tackle the GRE Issue essay because of your experience explaining how evidence supports your point . You’ll also likely be able to use at least one or two of the examples you’ve been writing about, even if you have to explain their support of your point of view in a different way than you’ve practiced.

Want to improve your GRE score by 7+ points?

Check out our best-in-class online GRE prep program . We guarantee your money back if you don't improve your GRE score by 7 points or more.

PrepScholar GRE is entirely online, and it customizes your prep program to your strengths and weaknesses . We also feature 2,000 practice questions , official practice tests, 150 hours of interactive lessons, and 1-on-1 scoring and feedback on your AWA essays.

Check out our 5-day free trial now:

body_preplanexamples

#3: Analyze Sample Essays

Scour the sample essays ETS has publicly released to understand at a deep level what is required for a 6-scoring GRE Issue essay. In addition to the essay briefly discussed in this article, perfect-scoring sample Issue essays can also be found in chapters 8 and 9 of The Official Guide to the GRE revised General Test (2nd Ed.) .

To get the most out of these exemplars of perfect essay scores, you should analyze these sample essays using the scoring rubric. Use the points we focused on above in the 4-vs.-6 rubric score comparison and the sample Issue essay breakdown as guidance to find specific ways the sample essays fulfill the rubric scoring guidelines. The essays in the Official GRE Guide also include reader commentary on the essays, which are good sources of further insight into the thought processes of essay raters.

The goal of performing these analyses of sample high-scoring GRE essays is for you to understand what makes the essays high-scoring and then be able to replicate this high level of essay writing in your own Issue essay on the GRE. This doesn’t mean that you should copy the exact words or phrases from the essays (that’s plagiarism, which is both wrong and against the GRE’s code of conduct), but it does mean you should observe the ways other students have successfully met the rubric requirements.

body_analyzeotheressays

#4: Leave Time to Revise Your Work

Sadly, the word processor you’ll use on the GRE has no autocorrect, no spellcheck, and no grammar check. Since you’re trying to type as much as possible in a timed situation, it’s very likely you’ll make some errors.

It’s fine to make a few small mistakes on your essay as long as the typos or other mistakes don’t make your essay difficult to understand. If there are systematic typos or grammatical errors, however, that will have a negative effect on your score , because it will obscure your logic and make it more difficult for the graders (human and computer) to understand your thinking.

Example 1: No editing, systematic errors

Choosing a college major based on the avilablility of jobs in the field is a foolish endaevor at best. There’s no guarantee that Just because there are a lot of positions open in the field when you choose your undergraduate major, it doesn’t necessarily follow: this will continue ot be the case after you graduate from college, or even when you’re looking for a job.

Example 2: Edited, minor errors remaining

Choosing a college based on the availability of jobs in the field is a foolish endaevor at best. Just because there are a lot of positions open in the field when you choose your undergraduate major, it doesn’t necessarily follow that this will continue ot be the case after you graduate from college, or even when you’re looking for a job.

OER Africa/Flickr

What’s Next?

After reading this article, you’ve gained some clarity on what kind of GRE Writing score you need to succeed, but how well do you need to do on the other sections of the test? Learn what makes a good (or a bad) GRE score with this article .

Looking to get more clarity into the whole essay-scoring process? We have a guide to how the GRE essay is scored that explains it from start to finish, including how computerized grading plays into your essay score.

Want even more in-depth analyses of high-scoring GRE essays? Then be sure to check out our article analyzing perfect- and high-scoring Issue and Argument essays .

Ready to improve your GRE score by 7 points?

template for gre issue essay

Author: Laura Staffaroni

Laura graduated magna cum laude from Wellesley College with a BA in Music and Psychology, and earned a Master's degree in Composition from the Longy School of Music of Bard College. She scored 99 percentile scores on the SAT and GRE and loves advising students on how to excel and fulfill their college and grad school dreams. View all posts by Laura Staffaroni

template for gre issue essay

Reach out to ask any questions about our services! Call or Text (973) 922-0818 or Email [email protected]

template for gre issue essay

PREP FOR SUCCESS

practice GRE Issue Essay notebook

How to write a perfect GRE Issue Essay: Tips, Tricks, and Examples

Gre essay topics.

  • Government and Power Example: “Governments must ensure that their major cities receive the financial support they need in order to thrive.”
  • Education Example: “Educational institutions have a responsibility to dissuade students from pursuing fields of study in which they are unlikely to succeed.”
  • Culture and Society Example: “The most effective way to understand contemporary culture is to analyze the trends of its youth.”

Main Structure of Your Essay

How to write your thesis statement, 10 best gre essay tips.

  • Make sure to use topic strong topic sentences to organize and introduce your paragraphs.
  • Use varied sentence structure.
  • Use transitional words and phrases such as: “however”, “therefore”, “consequence”, “on the contrary”, “while” “even though”, etc.
  • Use proper grammar. This is a must!
  • Use active voice when writing. For instance, instead of saying “The light bulb was invented by Thomas Edison”say “Thomas Edison invited the light bulb”.
  • Avoid using slang and cliches.
  • Instead of focusing on one single point, make sure you include a verity of examples. Don’t over-exhaust one example.
  • PROOFREAD – make sure you leave a few minutes at the end to proofread your writing. This is key.
  • Practice. You won’t become a better writer if you don’t write!
  • If you are still having trouble after reading this blog, get help! Hiring a tutor for even one or two sessions can boost your score tremendously

GRE Essay Directions

“The best way to teach is to praise positive actions and ignore negative ones.”

Write a response in which you discuss the extent to which you agree or disagree with the recommendation and explain your reasoning for the position you take. In developing and supporting your position, describe specific circumstances in which adopting the recommendation would or would not be advantageous and explain how these examples shape your position.

GRE Sample Essay

     The recommendation presents a view that I would agree is successful most of the time, but one that I cannot fully support due to the “all or nothing” impression it gives.      Certainly as an educator I agree fully that the best way to elicit positive response from students is to make use of students’ positive energy and then encourage actions that you would like to see repeated. It is human nature that we all want to be accepted and achieve on some level, and when people in authority provide feedback that we have done something well, the drive to repeat the action that was praised is bound to be particularly strong.      This blanket statement would obviously pay dividends in situations in which a teacher desires to have students repeat particular behaviors. For example, if an educator is attempting to teach students proper classroom etiquette, it would be appropriate to openly praise a student who raises his or her hand when wishing to speak or address the class. In such cases, the teacher may also help shape positive behaviors by ignoring a student who is trying to interject without approval from the teacher. In fact, the decision to ignore students who are exhibiting inappropriate behaviors of this type could work very well in this situation, as the stakes are not very high and the intended outcome can likely be achieved by such a method. However, it is important to note here that this tactic would only be effective in such a “low-stakes” situation, as when a student speaks without raising her hand first. As we will discuss below, ignoring a student who hits another student, or engages in more serious misbehaviors, would not be effective or prudent.      To expand on this point, it is important for teachers to be careful when working with the second half of this statement, only ignoring negative actions that are not serious. Take for instance a student who is misbehaving just by chatting with a fellow classmate. This student might not be presenting much of a problem and may be simply seeking attention. Ignoring the student might, in fact, be the best solution. Now assume the negative action is the improper administering of chemicals in a science experiment or the bullying of a fellow student. To ignore these negative actions would be absurd and negligent. Now you are allowing a problem to persist, one that could potentially lead to much bigger and more dangerous issues. In a more serious situation, addressing the negative actions quickly and properly could stop the problem it in its tracks. It is for reasons like this that I do not advocate the idea that a teacher can be successful by simply ignoring negative actions.      I do, however, greatly support the idea that the central focus of teaching should be to build on and encourage positive actions. However, the author’s all-encompasing statement leaves too many negative possibilities for the classroom. Perhaps a better way to phrase this statement would be to say, “The best way to teach is to praise positive actions and ignore negative ones that are not debilitating to class efficiency or the safety of any individual”.      Thus, in the original statement, there are indeed some good intentions, and there could be a lot of merit in adopting its basic principles. Data proves that positive support can substantially increase motivation and desire in students and contribute to positive achievements. In fact, most studies of teaching efficacy indicate that praising positive actions and ignoring negative ones can create a more stable and efficient classroom. It needs to be stressed, however, that this tool is only effective at certain levels of misbehavior. As mentioned above, when the behavior is precipitated by feelings of revenge, power or total self-worthlessness, this methodology will likely not work. It is likely to be very successful, however, when the drive behind the misbehavior is simple attention seeking. In many of these instances, if the teacher demonstrates clearly that inappropriate behavior does not result in the gaining of attention, students are more likely to seek attention by behaving properly. Should the student choose this path, then the ignoring has worked and when the positive behavior is exhibited, then the teacher can utilize the first part of the theory and support or praise this behavior. Now it is much more likely to be repeated. If the student does not choose this path and instead elects to raise the actions to a higher level that presents a more serious issue, then ignorance alone cannot work and other methods must be employed.      In conclusion, one can appreciate the credo expressed in this instance, but surely we all can see the potential error of following it through to the extreme.

CITATION: The above sample essay is adapted from the ETS website. For the direct source click here.

About the Author

template for gre issue essay

About our GRE Tutors

Prep For Success has a team of amazing GRE tutors, who have all scored in the 99th percentile. Additionally, all of them have been rigorously trained on all of our tips, tricks, and strategies to obtain a high score! Each tutor takes the time to understand your specific strengths and weaknesses so you don’t waste time in your studies and have the most effective study plan! Feel free to reach out to get assistance on the GRE or Graduate School Admissions.

Email this post to yourself to study later! Or share it with a friend studying for the GRE!

  • February 19, 2021
  • No Comments

template for gre issue essay

How to Write a Personal Statement

Harvard graduate on campus

MIT vs Harvard: Deciding the Best Fit for Your Academic Success

student-practicing-for-virtual-LSAT

Upcoming LSAT Changes: No More Logic Games Confirmed

Leave a reply cancel reply.

You must be logged in to post a comment.

  • Become a Tutor
  • Referral Program
  • Privacy Policy
  • Terms of Service
  • Refund/Cancelation/Reschedule Policy

Want to talk to a live Prep For Success representative that can answer all your questions? If so, reach out!

logo

Guidance from our top admission experts — for free!

bachelors

  • Admit Finder

Discover Past Admits, Gauge Your Chances!

  • Shortlist Builder

Personalized University Picks, Just a Click Away.

  • Course Finder

Navigate Global Courses Tailored for You

  • Scholarship Finder

Unlock Funding Opportunities Worldwide.

pro

Get tailored study abroad advice.

close

Sign in for exclusive content!

template for gre issue essay

Planning to study abroad?

close

Build your target shortlist and see your odds of getting into top schools with Ambitio's AI shortlist builder!

close

Heading Out Already?

Our Ivy League mentors and top admission experts can help with personalized tips to get you into your dream school

1 September 2023

9 minutes read

Nail the GRE Analytical Writing with Sample Issue Essays

author-avatar

Worried about the cost of Studying Abroad?

Sign up to access 25 game-changing scholarships that could cover your costs.

illustration

  • Introduction

The Graduate Record Examination (GRE) is a standardized test required by many graduate schools in the United States and other English-speaking countries.

One of the most challenging sections of the GRE is the analytical writing section, which consists of two tasks: the issue task and the argument task.

This blog will focus on the issue task, providing GRE issue essay examples, tips, and strategies to help you nail the GRE analytical writing section.

Stuck on How to Pick Your Ideal College?

Sign up to access your tailored shortlist and simplify finding your ideal college.

illustration

  • GRE Analytical Writing Section

The analytical writing section is the first part of the GRE and is designed to assess your critical thinking and analytical writing skills. It consists of two 30-minute tasks: the ‘Analyze an Issue’ task and the ‘Analyze an Argument’ task.

In the issue task, you are given a statement on a general issue, and you are required to write a response in which you discuss your views on the issue. You will need to provide reasons and examples to support your position.

The quality of your essay, your ability to present a clear and well-reasoned response, and your writing skills will all contribute to your GRE analytical writing score.

See how Successful Applications Look Like!

Access 350K+ profiles of students who got in. See what you can improve in your own application!

illustration

  • Preparing for the GRE Issue Essay

The issue essay requires you to take a position on an issue and to support your position with reasons and examples. It is important to practice writing essays on various topics to develop your writing skills and to understand the types of topics that may appear in the actual GRE exam.

Analyzing the Prompt

The first step in preparing for the issue essay is to analyze the prompt. The prompt will present a statement, and you will need to discuss the extent to which you agree or disagree with the statement.

It is important to carefully read and understand the prompt and to identify the key issues that need to be addressed in your essay.

Planning Your Response

Once you have analyzed the prompt, it is important to plan your response. Start by deciding on your position on the issue. Do you agree or disagree with the statement?

To what extent do you agree or disagree? Then, think about the reasons for your position and the examples that could be used to support your reasoning.

Writing the Essay

When writing the essay, start with an introduction that clearly states your position on the issue and provides a brief overview of the reasons for your position.

In the body of the essay, discuss your reasons and provide examples to support your position. Be sure to address the most compelling reasons and examples that could be used to challenge your position. Finally, in the conclusion, summarize your position and the reasons for your position.

get-pro

Start Your University Applications with Ambitio Pro!

Get Ambitio Pro!

Begin your journey to top universities with Ambitio Pro. Our premium platform offers you tools and support needed to craft standout applications.

Unlock Advanced Features for a More Comprehensive Application Experience!

get-pro

Start your Journey today

  • GRE Issue Essay Examples

Below are some sample issue essay topics and responses to help you prepare for the GRE analytical writing section.

Sample Issue Essay 1:

Prompt : “In today’s world, it is more important to work quickly and risk making mistakes than to work slowly and make sure that everything is perfect.”

Introduction : In today’s fast-paced world, there is often a trade-off between speed and accuracy. While it is important to work quickly to meet deadlines and stay competitive, it is also important to ensure that the work is of high quality and free of mistakes. I partially agree with the statement that it is more important to work quickly and risk making mistakes than to work slowly and make sure that everything is perfect. While speed is important in some situations, accuracy is also crucial, and it is important to strike a balance between the two.

Body : On one hand, working quickly is important in many situations. In today’s competitive business environment, companies often face tight deadlines and need to respond quickly to changes in the market. Being able to work quickly can lead to increased productivity, faster decision-making, and a competitive advantage. Additionally, in some situations, it is better to make a decision quickly, even if it is not perfect than to delay and miss out on an opportunity.

On the other hand, it is also important to ensure that the work is of high quality and free of mistakes. Making mistakes can lead to wasted time and resources, and can damage a company’s reputation. In some industries, such as healthcare or aviation, making mistakes can have serious consequences and can even be life-threatening.

Conclusion : In conclusion, while it is important to work quickly in some situations, it is also important to ensure that the work is of high quality and free of mistakes. It is important to strike a balance between speed and accuracy, and to consider the potential consequences of making mistakes. Ultimately, the best approach will depend on the situation and the potential consequences of making mistakes.

Sample Issue Essay 2:

Prompt : “The best way to solve environmental problems caused by consumer-generated waste is for towns and cities to impose strict limits on the amount of trash they will accept from each household.”

Introduction : Environmental problems caused by consumer-generated waste are a growing concern worldwide. Landfills are overflowing, and pollution from waste is impacting air and water quality. While imposing strict limits on the amount of trash accepted from each household may be part of the solution, I do not believe it is the best way to solve the problem. A more comprehensive approach that includes reducing, reusing, and recycling waste, as well as educating the public about the importance of reducing waste, is needed to effectively address the issue.

Body : Imposing strict limits on the amount of trash accepted from each household may encourage people to reduce their waste and recycle more. However, it may also lead to illegal dumping or to people finding other ways to dispose of their waste that may be even more harmful to the environment. Additionally, it does not address the root causes of the problem, which include overconsumption and the production of non-recyclable or non-biodegradable products.

A more comprehensive approach is needed to effectively address the issue. This could include implementing policies that encourage the production and use of recyclable or biodegradable products, promoting recycling and composting programs, and educating the public about the importance of reducing waste. Additionally, providing incentives for households and businesses to reduce their waste and recycle, as well as imposing penalties for illegal dumping, could also be part of the solution.

Conclusion : In conclusion, while imposing strict limits on the amount of trash accepted from each household may be part of the solution, it is not the best way to solve the problem. A more comprehensive approach that includes reducing, reusing, and recycling waste, as well as educating the public about the importance of reducing waste, is needed to effectively address the issue.

  • Tips for Writing a High-Scoring GRE Issue Essay
  • Analyze the Prompt : Make sure you understand the prompt and the issue it presents. Identify the key issues that need to be addressed in your essay.
  • Plan Your Response : Decide on your position on the issue and think about the reasons for your position and the examples that could be used to support your reasoning.
  • Write a Clear and Well-Reasoned Response : Start with an introduction that clearly states your position on the issue and provides a brief overview of the reasons for your position. In the body of the essay, discuss your reasons and provide examples to support your position. Be sure to address the most compelling reasons and examples that could be used to challenge your position. Finally, in the conclusion, summarize your position and the reasons for your position.
  • Practice, Practice, Practice : Practice writing essays on various topics to develop your writing skills and to understand the types of topics that may appear in the actual GRE exam.
  • Revise Your Essay : After writing your essay, take some time to revise it. Check for any grammatical or spelling errors, and make sure that your ideas are clearly and logically presented.

Understanding the GRE Issue Task Scoring Criteria

Understanding the criteria on which your essay will be evaluated can help you write a high-scoring essay. ETS, the organization that administers the GRE, evaluates the issue essay on the following criteria:

  • Articulation of Ideas : This criterion assesses the clarity and fluency with which you present your ideas. Your essay should be well-organized, with a clear and logical progression of ideas.
  • Development and Support : This criterion assesses the reasons and examples you provide to support your position on the issue. Your reasons should be well-developed, and your examples should be relevant and compelling.
  • Analytical Writing Skills : This criterion assesses your ability to analyze the issue, consider different perspectives, and present a well-reasoned and thoughtful response.
  • Control of Language : This criterion assesses the quality of your writing, including your vocabulary, sentence structure, and grammatical correctness.

To score high on the GRE issue essay, make sure to address all of these criteria in your essay.

Developing a Strong Thesis Statement

Your thesis statement is one of the most important parts of your essay. It should clearly state your position on the issue and provide a brief overview of the reasons for your position.

Your thesis statement should be specific and debatable, and it should provide a roadmap for your reader. Here are some tips for developing a strong thesis statement:

  • Take a Clear Position : Your thesis statement should clearly state your position on the issue. Avoid vague or neutral statements that do not take a clear position.
  • Be Specific : Your thesis statement should be specific and focused. Avoid broad or general statements that do not clearly articulate your position.
  • Provide a Roadmap : Your thesis statement should provide a brief overview of the main points that you will discuss in your essay. This will help your reader understand the structure of your essay and the main ideas that you will be discussing.

Addressing Opposing Views

It is important to address opposing views in your essay. This shows that you have considered different perspectives on the issue and helps to strengthen your argument. Here are some tips for addressing opposing views in your essay:

  • Be Respectful : Acknowledge the validity of opposing views, even if you do not agree with them. Avoid dismissive or derogatory language.
  • Be Fair : Present opposing views accurately and fairly. Do not misrepresent or distort opposing views to make your own argument seem stronger.
  • Provide a Counterargument : After presenting an opposing view, provide a counterargument that explains why you do not agree with that view and why your position is more persuasive.

Using Evidence Effectively

Using evidence effectively is crucial for writing a persuasive essay. Your evidence should be relevant, compelling, and should directly support your thesis statement. Here are some tips for using evidence effectively in your essay:

  • Be Specific : Provide specific examples and details to support your thesis statement. Avoid vague or general statements that do not directly support your argument.
  • Be Relevant : Make sure that the evidence you provide is directly relevant to your thesis statement and the main points of your essay.
  • Be Persuasive : Use compelling evidence that persuades your reader to accept your position on the issue. This could include facts, statistics, anecdotes, or expert opinions.

Crafting a Strong Conclusion

Your conclusion is the final opportunity to persuade your reader to accept your position on the issue. It should summarize your main points, restate your thesis in a new way, and provide a closing thought that leaves a lasting impression on your reader. Here are some tips for crafting a strong conclusion:

  • Summarize Your Main Points : Briefly summarize the main points of your essay. This helps to reinforce your argument and reminds your reader of the key reasons for your position.
  • Restate Your Thesis : Restate your thesis in a new way. This helps to reinforce your argument and provides a sense of closure to your essay.
  • Provide a Closing Thought : End your essay with a closing thought that leaves a lasting impression on your reader. This could be a call to action, a prediction about the future, or a final insight about the issue.
  • Practice is Key

Practicing writing essays on various topics is crucial for developing your writing skills and for understanding the types of topics that may appear in the actual GRE exam.

Practice writing essays on different topics, and seek feedback from others to improve your writing. Additionally, you can find sample issue essay prompts on the ETS website or in official GRE prep books.

Practice writing essays on these prompts to familiarize yourself with the types of topics that may appear in the actual exam and to develop your ability to write a well-reasoned and thoughtful response.

The GRE issue essay is an important part of the GRE analytical writing section. By understanding the scoring criteria, developing a strong thesis statement, addressing opposing views, using evidence effectively, and crafting a strong conclusion, you can write a high-scoring GRE issue essay.

Remember to practice writing essays on various topics to develop your writing skills and to familiarize yourself with the types of topics that may appear in the actual GRE exam. With practice and preparation, you can nail the GRE analytical writing section and achieve a high score.

What is the GRE analytical writing section?

The GRE analytical writing section is the first part of the GRE and is designed to assess your critical thinking and analytical writing skills. It consists of two 30-minute tasks: the ‘Analyze an Issue’ task and the ‘Analyze an Argument’ task.

How is the GRE issue essay scored?

The GRE issue essay is scored on a scale of 0 to 6, with 0 being the lowest score and 6 being the highest score. ETS evaluates the issue essay on four criteria: articulation of ideas, development, and support, analytical writing skills, and control of language.

What are some tips for writing a high-scoring GRE issue essay?

Some tips for writing a high-scoring GRE issue essay include understanding the scoring criteria, developing a strong thesis statement, addressing opposing views, using evidence effectively, and crafting a strong conclusion.

How can I practice for the GRE issue essay?

Practice writing essays on various topics to develop your writing skills and to understand the types of topics that may appear in the actual GRE exam. Additionally, you can find sample issue essay prompts on the ETS website or in official GRE prep books. Practice writing essays on these prompts to familiarize yourself with the types of topics that may appear in the actual exam and to develop your ability to write a well-reasoned and thoughtful response.

Spread the Word!

Share across your social media if you found it helpful

logo

Table of Contents

  • • Introduction
  • • GRE Analytical Writing Section
  • • Preparing for the GRE Issue Essay
  • • GRE Issue Essay Examples
  • • Tips for Writing a High-Scoring GRE Issue Essay
  • • Practice is Key
  • • Conclusion

Build your profile to get into top colleges

Phone Number

What level are you targetting

back

Almost there!

Just enter your OTP, and your planner will be on its way!

Code sent on

Resend OTP (30s)

whatsapp

Your Handbook Is Waiting on WhatsApp!

Please have a look, and always feel free to reach out for any detailed guidance

whatsapp

Click here to download

Meanwhile check out your dashboard to access various tools to help you in your study abroad journey

get-pro

Recent Blogs

template for gre issue essay

Esade MBA GMAT Guide: Navigating Admission Process at Esade Business School in Barcelona

template for gre issue essay

Purdue University’s Engineering Management: GRE, MEM, and MS in Engineering – Insights into Class Profile and IELTS Requirements

template for gre issue essay

Emory Goizueta MBA Program: A Comprehensive Guide for Prospective MBA Applicants and Admission Insights

Find your Dream school now⭐️

Welcome! Let's Land Your Dream Admit.

Let us make sure you get into the best!

dropdown-icon

  • 2024 Winter
  • 2024 Spring
  • 2024 Summer

Enter verification code

Code was sent to

logo

  • Our Experts

Connect with us on our social media

template for gre issue essay

GRE Analytical Writing Sample Essays

 alt=

Introduction

The GRE ® essay section is also referred to as the AWA or the GRE ® Analytical Writing Assessment which experts believe is one of the most neglected sections of the GRE ® test. Most test-takers believe that they can master the section in a few days at the most. A couple of GRE ® sample essays should be sufficient. The outcome is not desirable with an average global score of 4.0 with the Indian score even lower.

GRE ® Essay Sections

This Analytical Writing Assessment (AWA) portion is made up of two parts:

  • Issue essays
  • Argument essays

It is aimed at measuring the following:

  • Communicate intricate thoughts distinctly and effectually
  • Back notions with appropriate explanations and illustrations
  • Scrutinize claims and supplementary proof
  • Maintain a well-focused, articulate argument
  • Command the components of standard written English

The structure of the GRE ® essay has been designed to test your ability to write a cogent thesis statement that you must defend over the course of several paragraphs.

  • You are allocated a time limit of 30 minutes to complete each section. The is the first section and the structure of the test does not allow you to skip it and come back to it later.
  • The GRE ® Essay expects you to analyze your critical thinking abilities.
  • This allocated topic is usually a statement that is rather broad in nature.
  • You will be evaluated for your capabilities to reason analytically and to debate the given topic from your perspective.
  • You will need to support your viewpoint with appropriate instances and substantiation and structure your answer according to the precise guidelines that will be associated with the task.
  • It is difficult to understand the requirements of this task without first going through relevant GRE ® sample essays.

How to use this Guide for AWA Passage Writing?

  • There are two ways by which you can make use of this guide not just to improve your essay but also to master essay grading.
  • Begin by taking a look at some of these perfectly scored sample essays.
  • Remember that these samples that you go through will help you understand sentence structures, body paragraphs, etc. You should never ever think of copy-pasting direct sentences when you are writing your test as that will be considered as plagiarism.
  • Use the guide in a manner that helps to incorporate features that help to highlight your position on the issue.
  • The secondary objective of using this guide is to overcome your writing weaknesses in conjunction with essay grading.
  • To ensure that you score high, begin the process with the rubrics for the Issue and Argument Tasks and subsequently zero in on the section that you find most difficult to meet.
  • The idea is for you to identify the areas that you have trouble with the most. By taking adequate GRE ® prep, and going through samples and essay responses from experts, you will be able to overcome these difficulties.

What’s new?

Knowing how to get that perfect score is important. Find out more about GRE ® Score and the grading system.

How does the GRE ® essay work?

  • With two parts to the GRE ® Analytical Writing Assessment (AWA), you are allotted 30 minutes for each essay.
  • This is aimed to test your ability to write a thesis statement that is cogent, which you should be able to support with adequate evidence over the course of the essay.

What is the difference between the Issue and Argument essays?

  • A sample topic reads like – “It could be argued that the most important technological breakthroughs have happened by chance and through surprise discoveries. However, others argue that deliberate, well-planned research with specific goals is the only way to ensure technological advancement.
  • Write a response in which you discuss the extent to which you agree or disagree with the statement and explain your reasoning for the position you take. In developing and supporting your position, you should consider ways in which the statement might or might not hold true and explain how these considerations shape your position.”
  • You are expected to respond by analyzing this general statement by taking a stance over a matter that is understandably complex.
  • You may get similar topics encompassing several different areas such as politics, education, or culture.
  • A sample topic reads – “SuperCorp recently moved its headquarters to Corporateville. The recent surge in the number of homeowners in Corporateville proves that Corporateville is a superior place to live then Middleburg, the home of SuperCorp’s current headquarters. Moreover, Middleburg is a predominantly urban area and according to an employee survey, SuperCorp has determined that its workers prefer to live in an area that is not urban. Finally, Corporateville has lower taxes than Middleburg, making it not only a safer place to work but also a cheaper one. Therefore, Supercorp clearly made the best decision.

Write a response in which you examine the stated and/or unstated assumptions of the argument. Be sure to explain how the argument depends on the assumptions and what the implications are if the assumptions prove unwarranted.”

  • In contrast to the Issue essay, the Argument essay will ask the author to dissect the logic behind the stance or position taken by him or her.

Where can I find sample topics?

  • You can log on to the ETS website where they publish pools of Issue and Argument essay topics.
  • These topics have been taken from previous tests.

How are the essays scored?

  • There is an initial human grader of the essay who has received rigorous training to qualify to be a grader.
  • He or she is typically a university literature/writing professor.
  • Additionally, there is an ‘E-rater’ which is an automated essay grader.
  • This system ensures that the human grader has justifiably scored the essays without bias or prejudice.
  • The final score is an average of the two scores and the result rounded off to fit the half point scale.

How does the GRE ® AWA scoring range work?

  • Take a look at the scoring guidelines that will help you understand where you stand with the score that you have received.Compare the essays that you have written with sample essays in order for you to get a sense of what score you might receive for them.
  • Compare the essays that you have written with sample essays in order for you to get a sense of what score you might receive for them.

Is there anywhere I can get my essays graded?

  • You can have someone that you trust give you their honest feedback.
  • Alternatively, ETS offers a service to grade your sample GRE ® AWA essays but that does not give you any feedback, only the score.
  • You can also access forums where you can upload your sample essays to get insights and analysis.

What do the graders look for?

  • When your essays are graded, the three key pillars that graders look for that determine your score are clarity, coherency, and cogency.
  • You must communicate your ideas as clearly as you can which should be logically connected to one another as you transition between sentences and paragraphs.
  • Whatever be your claims, it should be supported by sufficient evidence and examples that are sustainable.
  • Style of writing is important which means you should avoid essay with choppy sentences, bad grammar, misspellings and unsophisticated vocabulary.
  • A typical grader takes 30 seconds to score your essay if he or she is satisfied with clear organization of your information, check if your paragraphs start with a topic sentence and flow into specific examples that support your analysis.

How long does my essay have to be?

  • The substance in your essay is of paramount importance as long as it has been clearly bifurcated into a five-paragraph format including an introduction, three body paragraphs, and a conclusion.
  • Keep a lookout for your paragraph lengths as long paragraphs minus logical matter will not grab you better scores.

How do I practice for GRE ® Analytical Writing?

  • Practice makes it perfect and it is no different for the GRE ® Analytical Writing Assessment.
  • Get a better command and facility on the language by writing more.
  • Create an outline and brainstorm on your position and think of a relevant example to support your claim.
  • Set aside time to edit your practice essays.
  • Look through tons of approved sample essays and correlate it with the grades that they have received.
  • While grammar is not of top priority as clarity of thought, it still holds weight and must be given its due importance in the scheme of things.

How do I improve my grammar and style?

  • You can improve your grammar by going through books such as William Zinsser’s On Writing Well.
  • To improve on your writing style, a book that comes highly recommended is Strunk and White’s Elements of Style.
  • You may consult other resources to complement these books and help in an overall improvement of grammar and style.

Are there any sample essays I can read?

  • Sample Issue Essays from ETS
  • Sample Argument Essays from ETS
  • The essays on the ETS website have been written by students.
  • You can use these essays as benchmarks to assess your own levels of writing.

GRE ® Sample Essays for the Issue Task

ETS provides brief information about an issue of common interest and tells the test taker to evaluate and analyze it. The candidate is required to essentially develop a sound argument for the issue and support it with examples.

ETS Essay Pool for Issue Essays (1 to 10):

The GRE ® Issue essay is similar in structure to the classic 5-paragraph short essay. You can go through the following links for familiarizing yourself with GRE ® sample essays pertaining to the Issue task.

  • GRE ® Issue Essay-1: "We learn through direct experience; to accept a theory without experiencing it is to learn nothing at all."
  • GRE ® Issue Essay-2: "Laws should not be rigid or fixed. Instead, they should be flexible enough to take account of various circumstances, times, and places."
  • GRE ® Issue Essay-3: "People are too quick to take action; instead, they should stop to think of the possible consequences of what they might do."
  • GRE ® Issue Essay-4: "It is possible to pass laws that control or place limits on people's behavior, but legislation cannot reform human nature. Laws cannot change what is in people's hearts and minds."
  • GRE ® Issue Essay-5: "Success in any realm of life comes more often from taking chances or risks than from careful and cautious planning."
  • GRE ® Issue Essay-6: "Originality does not mean thinking something that was never thought before; it means putting old ideas together in new ways."
  • GRE ® Issue Essay-7: "It is always an individual who is the impetus for innovation; the details may be worked out by a team, but true innovation results from the enterprise and unique perception of an individual."
  • GRE ® Issue Essay-8: "The study of an academic discipline alters the way we perceive the world. After studying the discipline, we see the same world as before, but with different eyes."
  • GRE ® Issue Essay-9: "If people disregard the great works of the past, it is because these works no longer answer the needs of the present."
  • GRE ® Issue Essay-10: "As we acquire more knowledge, things do not become more comprehensible, but more complex and more mysterious."

GRE ® Sample Essays for the Argument Task

The GRE ® Argument Essay asks you to change perspective from the one you had for the Issue Essay. Your essay should be about 5-6 paragraphs in which you will criticize an argument, describe how it could be improved, and reiterate that it is overall weak and unconvincing.

ETS Essay Pool for Argument Essays (1 to 10):

Take a look at the following sample argument essay topics:

  • GRE ® Argument Essay-1: "Seven years ago, homeowners in nearby Brookville community adopted a set of restrictions on how the community's yards should be landscaped and what colors the exteriors of homes should be painted. Since then, average property values have tripled in Brookville. In order to raise property values in Deerhaven Acres, we should adopt our own set of restrictions on landscaping and house painting."
  • GRE ® Argument Essay-2: "Of the two leading real estate firms in our town, Adams Realty, and Fitch Realty, Adams is clearly superior. Adams has 40 real estate agents. In contrast, Fitch has 25, many of whom work only part-time. Moreover, Adams' revenue last year was twice as high as that of Fitch and included home sales that averaged $168,000, compared to Fitch's $144,000. Homes listed with Adams sell faster as well: ten years ago, I listed my home with Fitch and it took more than four months to sell; last year when I sold another home, I listed it with Adams, and it took only one month. Thus, if you want to sell your home quickly and at a good price, you should use Adams."
  • GRE ® Argument Essay-3: "A jazz music club in Monroe would be a tremendously profitable enterprise. Currently, the nearest jazz club is 65 miles away; thus, our proposed club, the C Note, would have the local market all to itself. Plus, jazz is extremely popular in Monroe: over 100,000 people attended Monroe's jazz festival last summer, several well-known jazz musicians live in Monroe, and the highest-rated radio program in Monroe is 'Jazz Nightly,' which airs every weeknight. Finally, a nationwide study indicates that the typical jazz fan spends close to $1,000 per year on jazz entertainment. It is clear that the C Note cannot help but make money."
  • GRE ® Argument Essay-4: "Mesa Foods, a manufacturer of snack foods that currently markets its products within a relatively small region of the country, has strong growth potential. Mesa enjoyed a 20 percent increase in profits last year, and its best-selling product, Diabolique Salsa, has had increased sales over each of the past three years. Since Omni Inc. is interested in reaching 14-to-25-year-olds, the age group that consumes the most snack food, we should buy Mesa Foods, and concentrate in particular on marketing Diabolique Salsa throughout the country."
  • GRE ® Argument Essay-5: "During the past year, Alta Manufacturing had thirty percent more on-the-job accidents than nearby Panoply Industries, where the work shifts are one hour shorter than ours. Experts believe that a significant contributing factor in many on-the-job accidents is fatigue and sleep deprivation among workers. Therefore, to reduce the number of on-the-job accidents at Alta and thereby increase productivity, we should shorten each of our three work shifts by one hour so that our employees will get adequate amounts of sleep."
  • GRE ® Argument Essay-6: "Previous experience has shown that our stores are most profitable in areas where residents are highly concerned with leading healthy lives. We should, therefore, build our next new store in Plainsville, which has many such residents. Plainsville merchants report that sales of running shoes and exercise clothing are at all-time highs. The local health club, which nearly closed five years ago due to lack of business, has more members than ever, and the weight training and aerobics classes are always full. We can even anticipate a new generation of customers: Plainsville's schoolchildren are required to participate in a 'fitness for life' program, which emphasizes the benefits of regular exercise at an early age."
  • GRE ® Argument Essay-7: “In Megalopolis, the number of law school graduates who went to work for large, corporate firms declined by 15 percent over the last three years, whereas an increasing number of graduates took jobs at small, general practice firms. Even though large firms usually offer much higher salaries, law school graduates are choosing to work for smaller firms most likely because they experience greater job satisfaction at smaller firms. In a survey of first-year students at a leading law school, most agreed with the statement that earning a high salary was less important to them than job satisfaction. This finding suggests that the large, corporate firms of Megalopolis will need to offer graduates more benefits and incentives and reduce the number of hours they must work.”
  • GRE ® Argument Essay-8: "Given that the number of people in our country with some form of arthritis is expected to rise from 40 million to 60 million over the next twenty years, pharmaceutical companies that produce drugs for the treatment of arthritis should be very profitable. Many analysts believe that in ten years Becton Pharmaceuticals, which makes Xenon, the best-selling drug treatment for arthritis, will be the most profitable pharmaceutical company. But the patent on Xenon expires in three years, and other companies will then be able to produce a cheaper version of the drug. Thus, it is more likely that in ten years the most profitable pharmaceutical company will be Perkins Pharmaceuticals, the maker of a new drug called Xylan, which clinical studies show is preferred over Xenon by seven out of ten patients suffering from the most extreme cases of arthritis."
  • GRE ® Argument Essay-9: "In the next mayoral election, residents of Clearview should vote for Ann Green, who is a member of the Good Earth Coalition, rather than for Frank Braun, a member of the Clearview town council, because the current members are not protecting our environment. For example, during the past year, the number of factories in Clearview has doubled, air pollution levels have increased, and the local hospital has treated 25 percent more patients with respiratory illnesses. If we elect Ann Green, the environmental problems in Clearview will certainly be solved."
  • GRE ® Argument Essay-10: "Two years ago, our consultants predicted that West Egg's landfill, which is used for garbage disposal, would be completely filled within five years. During the past two years, however, town residents have been recycling twice as much aluminum and paper as they did in previous years. Next month the amount of material recycled should further increase since charges for garbage pickup will double. Furthermore, over ninety percent of the respondents to a recent survey said that they would do more recycling in the future. Because of our residents' strong commitment to recycling, the available space in our landfill should last for considerably longer than predicted."

Issue Essay 1: Technology and Human Ingenuity

The topic assigned here is: “As people rely more and more on technology to solve problems, the ability of humans to think for themselves will surely deteriorate.

Discuss the extent to which you agree or disagree with the statement and explain your reasoning for the position you take. In developing and supporting your position, you should consider ways in which the statement might or might not hold true and explain how these considerations shape your position.”

  • The essay on this specific issue prompts the author to take a position instead of deterring their ability to reason about how technology will stimulate the human race to reach greater goals in life.
  • The sample essay looks at a number of possible avenues by which it strikes the right chord with the GRE ® essay rubric criteria to attain that perfect score.
  • The primary rubric criteria are the way in which the author adopts an insightful and clear stance on the given issue in the essay.
  • Over the entire course of the essay, the author articulates his or her position about the possibilities of embracing new technology as opposed to being fearful of it.
  • Paragraph 1: The author recognizes the fact that “technology has revolutionized the world.”
  • Paragraph 2: The author elucidates the thinking behind the declaration in the prompt - “The assumption is that an increased reliance on technology negates the need for people to think creatively to solve previous quandaries”.
  • Paragraph 3: The author refutes the rationale that was deliberated in paragraph 2, writing that “reliance on technology does not necessarily preclude the creativity that marks the human species.”
  • Paragraph 4: The author progresses with her counterclaim one step further, stating that “technology frees the human imagination.”
  • Paragraph 5: The author additionally cultivates the notion from Paragraph 4, stating “By increasing our reliance on technology, impossible goals can now be achieved.”
  • Paragraph 6: This final paragraph successfully ends the essay with a fully expressed thesis that also computes to what went before: “There is no need to retreat to a Luddite attitude to new things, but rather embrace a hopeful posture to the possibilities that technology provides for new avenues of human imagination.”
  • The author’s clear-cut rationalizations of her opinion and logic augment the lucidity of her position, while the nuanced content of the position itself establishes perception into the issue.

Issue Essay 2: Cooperation Vs. Competition

The topic assigned here is: “The best way for a society to prepare its young people for leadership in government, industry, or other fields is by instilling in them a sense of cooperation, not competition.”

Write a response in which you discuss the extent to which you agree or disagree with the claim. In developing and supporting your position, be sure to address the most compelling reasons or examples that could be used to challenge your position.

  • The author of this sample Issue essay has decided on a position on the issue by siding with the cooperation and not competition which is the preferred value to inculcate in young adults in preparation for government
  • The last line of the first paragraph “I would have to agree that the best way to prepare young people for leadership roles is to instill in them a sense of cooperation,” makes a clear declaration that absolutely fulfills the first part of the criteria.
  • The conclusion, particularly the last two sentences reiterates this stance.
  • Just being clear on your stance alone will not help you achieve that perfect score as you need to complement it with demonstrable insights into the core issue.
  • In the second part, the author takes on a two-pronged approach in order to answer the essay question as opposed to just clarifying how cooperativeness spearheads optimistic results in government, industry, and other fields and competitiveness results in negative consequences.
  • In the opening and closing paragraphs, the author makes this position even more categorical.
  • The following rubric criteria deal with how successfully the author fosters his standpoint with examples and reasoning, for example, by reviewing the downsides of competition.
  • Opening with general perceptions of human behaviors at school and the office to present his argument, the author then deftly segues into precise instances of competitiveness gone bad.
  • Each illustration is used by the author to drive his point down deeper with a final round off that competitiveness can help people only up to a certain point, but the price is that it is also “damaging and in due course quite disparaging.”
  • The logical interlinking of notions through paragraphs is the third parameter that makes this a perfectly scoring essay.
  • The manner in which the paragraphs are connected, the core focus still remains on the issue.
  • The author’s skill of using accurate language to communicate precise meaning epitomizes the author’s grasp of the language.

Argument Essay 1: Mason City Riverside Recreation

The topic assigned here is: “In surveys, Mason City residents rank water sports (swimming, boating, and fishing) among their favorite recreational activities. The Mason River flowing through the city is rarely used for these pursuits, however, and the city park department devotes little of its budget to maintaining riverside recreational facilities. For years there have been complaints from residents about the quality of the river’s water and the river’s smell. In response, the state has recently announced plans to clean up Mason River. The use of the river for water sports is therefore sure to increase. The city government should for that reason devote more money in this year’s budget to riverside recreational facilities.

  • The outcome of the survey is binding and demonstrative.
  • The explanation of why Mason River is not being used is by reason of its smell and contamination.
  • Getting rid of the contamination in the river will help free you from the smell thus leading to more residents using it.
  • The author recognizes the supposition that complaints point to countless people wanting to use the river and scrutinizes it by interpretation across potential situations other than the one exhibited in the prompt.
  • The understanding comes from the information that the exact opportunities deliberated by the author are exceedingly believable alternative clarifications for the facts that would transform the validity of the prompt’s assumption.
  • The outcome of her assessment concludes that there are unsatisfactory data to back the theory that Mason River is not used due to its smell and contamination.
  • The author of this sample GRE ® essay accomplishes the prerequisites of a textbook scoring Argument essay is by delivering wide-ranging support for each of her key points.
  • All through the essay, the author is able to illuminate accurately why every single assumption made is challenging by sourcing instances that exactly validate her argument.
  • What makes this sample Argument essay achieve a perfect score is how it has been organized logically, with clear transitions between ideas.
  • The author of this GRE ® essay sample is able to meet the first part of this requirement with a simple five-paragraph organizational structure: an introduction, one paragraph for each assumption discussed, and a conclusion.
  • Additionally, an Argument essay must be detailed and actual in its argument of notions, with minimum errors that the author successfully met with using purposeful language to efficiently and clearly get her point across.

Argument Essay 2: Super Screen Movie Advertising

The topic assigned here is: “According to a recent report from our marketing department, during the past year, fewer people attended Super Screen-produced movies than in any other year. And yet the percentage of positive reviews by movie reviewers about specific Super Screen movies actually increased during the past year. Clearly, the contents of these reviews are not reaching enough of our prospective viewers. Thus, the problem lies not with the quality of our movies but with the public’s lack of awareness that movies of good quality are available. Super Screen should, therefore, allocate a greater share of its budget next year to reaching the public through advertising.”

Write a response in which you discuss what questions would need to be answered in order to decide whether the recommendation and the argument on which it is based are reasonable. Be sure to explain how the answers to these questions would help to evaluate the recommendation.”

  • The first aspect of the essay examines how it successfully the content recognizes and assesses the portions of the argument that bears a close resemblance to the demands of the task.
  • As part of the conclusion, there are many queries that Super Screen is answerable to prior to making use of this advertising director’s plan.
  • There is a need to closely look into the actual numbers for viewership and positive reviews and ascertain the relationship that their target audience has with movie reviewers and establish how their target audience feels about their movies.
  • The author strikes on the three key points that should be taking into consideration prior to reaching an agreement with the advertising director’s plan: viewer and review numbers, audience reactions to reviews, and whether or not reviews are a useful metric by which to measure movie success.
  • An example that the author puts forward in relation to a particular argument can be found in the third paragraph of this GRE ® essay sample.
  • The paragraph begins by asserting the question that requires an answer – “What the number of positive reviews was and how it compared to pass reviews?”
  • Subsequently, post this preliminary recognition of the question, the author also justifies how responding this question would have an influence on the effectiveness of the recommendation: “If the increase in positive reviews was from 1% to 2%, allocating more money to advertising to emphasize this fact is likely to have less impact than if the money were instead budgeted towards improving film quality.”
  • The author of the GRE ® essay sample fulfills the requirement of sustaining every question she elevates the argument in the prompt by presenting how its reaction would shape the recommendation.
  • The author has also developed and connected notions in a clear and logical fashion.
  • The organization of this GRE ® argument essay sample facilitates in accomplishing this by steering the author’s views into an introduction, four body paragraphs, and a conclusion.
  • A perfect-scoring Argument essay must make use of complex and precise language, with few errors that the author of this essay has achieved.

Introduction – GRE ® Analytical Writing Sample Essays

Just like your verbal and Quant sections, you need tons of practice to master the essay section too. Go through these GRE ® sample AWA and response essays that will help test your ability.

Essay Response Score 6

The statement linking technology negatively with free thinking plays on the recent human experience over the past century. Surely there has been no time in history where the lived lives of people have changed more dramatically. A quick reflection on a typical day reveals how technology has revolutionized the world. Most people commute to work in an automobile that runs on an internal combustion engine. During the workday, chances are high that the employee will interact with a computer that processes the information on silicon bridges that are .09 microns wide. Upon leaving home, family members will be reached through wireless networks that utilize satellites orbiting the earth. Each of these common occurrences could have been inconceivable at the turn of the 19th century.

The statement attempts to bridge these dramatic changes to a reduction in the ability for humans to think for themselves. The assumption is that increased reliance on technology negates the need for people to think creatively to solve previous quandaries. Looking back at the introduction, one could argue that without a car, computer, or mobile phone, the hypothetical worker would need to find alternate methods of transport, information processing, and communication. Technology short circuits this thinking by making the problems obsolete.

However, this reliance on technology does not necessarily preclude the creativity that marks the human species. The prior examples reveal that technology allows for convenience. The car, computer, and phone all release additional time for people to live more efficiently. This efficiency does not preclude the need for humans to think for themselves. In fact, technology frees humanity to not only tackle new problems but may itself create new issues that did not exist without technology. For example, the proliferation of automobiles has introduced a need for fuel conservation on a global scale. With increasing energy demands from emerging markets, global warming becomes a concern inconceivable to the horse-and-buggy generation. Likewise, dependence on oil has created nation-states that are not dependent on taxation, allowing ruling parties to oppress minority groups such as women. Solutions to these complex problems require the unfettered imaginations of maverick scientists and politicians.

In contrast to the statement, we can even see how technology frees the human imagination. Consider how the digital revolution and the advent of the internet have allowed for an unprecedented exchange of ideas. WebMD, a popular internet portal for medical information, permits patients to self-research symptoms for a more informed doctor visit. This exercise opens pathways of thinking that were previously closed off to the medical layman. With increased interdisciplinary interactions, inspiration can arrive from the most surprising corners. Jeffrey Sachs, one of the architects of the UN Millenium Development Goals, based his ideas on emergency care triage techniques. The unlikely marriage of economics and medicine has healed tense, hyperinflation environments from South America to Eastern Europe.

This last example provides the most hope in how technology actually provides hope for the future of humanity. By increasing our reliance on technology, impossible goals can now be achieved. Consider how the late 20th century witnessed the complete elimination of smallpox. This disease had ravaged the human race since prehistorical days, and yet with the technology of vaccines, free-thinking humans dared to imagine a world free of smallpox. Using technology, battle plans were drawn out, and smallpox was systematically targeted and eradicated.

Technology will always mark the human experience, from the discovery of fire to the implementation of nanotechnology. Given the history of the human race, there will be no limit to the number of problems, both new and old, for us to tackle. There is no need to retreat to a Luddite attitude to new things, but rather embrace a hopeful posture to the possibilities that technology provides for new avenues of human imagination.

  • Rater Commentary for Essay Response — Score 6
  • The instances are convincing, have been developed with careful thought are logically aligned and well supported.
  • The transitions within ideas and paragraphs are smooth.
  • With a complex and varied sentence structure, the essay adheres to all standard norms of written English, i.e., grammar, usage, and mechanics.
  • Even though there are periodic errors visible, the essay still meets all the requirements thus attaining 6 which is the top GRE ® score for analytical writing.

Essay response score 5

Surely many of us have expressed the following sentiment, or some variation on it, during our daily commutes to work: "People are getting so stupid these days!" Surrounded as we are by striding and strident automatons with cell phones glued to their ears, PDA's gripped in their palms, and omniscient, omnipresent CNN gleaming in their eyeballs, it's tempting to believe that technology has isolated and infantilized us, essentially transforming us into dependent, conformist morons best equipped to sideswipe one another in our SUV's.

Furthermore, hanging around with the younger, pre-commute generation, whom tech-savviness seems to have rendered lethal, is even less reassuring. With "Teen People" style trends shooting through the air from tiger-striped PDA to zebra-striped PDA, and with the latest starlet gossip zipping from juicy Blackberry to teeny, turbo-charged cell phone, technology seems to support young people's worst tendencies to follow the crowd. Indeed, they have seemingly evolved into intergalactic conformity police. After all, today's tech-aided teens are, courtesy of authentic, hands-on video games, literally trained to kill; courtesy of chat and instant text messaging, they have their own language; they even have tiny cameras to efficiently photo-document your fashion blunders! Is this adolescence, or paparazzi terrorist training camp?

With all this evidence, it's easy to believe that tech trends and the incorporation of technological wizardry into our everyday lives have served mostly to enforce conformity, promote dependence, heighten consumerism and materialism, and generally create a culture that values self-absorption and personal entitlement over cooperation and collaboration. However, I argue that we are merely in the inchoate stages of learning to live with technology while still loving one another. After all, even given the examples provided earlier in this essay, it seems clear that technology hasn't impaired our thinking and problem-solving capacities. Certainly it has incapacitated our behavior and manners; certainly, our values have taken a severe blow. However, we are inarguably more efficient in our badness these days. We're effective worker bees of ineffectiveness!

If technology has so increased our sense of self-efficacy that we can become veritable agents of the awful, virtual CEO's of selfishness, certainly it can be beneficial. Harnessed correctly, technology can improve our ability to think and act for ourselves. The first challenge is to figure out how to provide technology users with some direly-needed direction.

  • Rater Commentary for Essay Response — Score 5
  • The strengths and weaknesses of this essay are clearly evident in the language used.
  • While some of the flowery representations do have a powerful impact, at times the descriptions are awkwardly placed, with the comparisons often being stressed.
  • The author has used complex vocabulary and syntax consistently – “Surrounded as we are by striding and strident automatons with cell phones glued to their ears, PDA's gripped in their palms, and omniscient, omnipresent CNN gleaming in their eyeballs, it's tempting to believe..."
  • Too much reliance on abstractions in the essay has not borne the desired effect as it lacks appropriate reasoning.
  • On the whole, the essay demonstrates credible ideas with examples with thoughtful analysis, taking an overall complex approach to the issue, thus earning a score 5.

Essay Response Score 4

In all actuality, I think it is more probable that our bodies will surely deteriorate long before our minds do in any significant amount. Who can't say that technology has made us lazier, but that's the keyword, lazy, not stupid? The ever-increasing amount of technology that we incorporate into our daily lives makes people think and learn every day, possibly more than ever before. Our abilities to think, learn, philosophize, etc. may even reach limits never dreamed of before by average people. Using technology to solve problems will continue to help us realize our potential as a human race.

If you think about it, using technology to solve more complicating problems gives humans a chance to expand their thinking and learning, opening up whole new worlds for many people. Many of these people are glad for the chance to expand their horizons by learning more, going to new places, and trying new things. If it wasn't for the invention of new technological devices, I wouldn't be sitting at this computer trying to philosophize about technology. It would be extremely hard for children in many poorer countries to learn and think for themselves without the invention of the internet. Think what an impact the printing press, a technologically superior machines at the time, had on the ability of the human race to learn and think.

Right now we are seeing a golden age of technology, using it all the time during our everyday lives. When we get up there's instant coffee and the microwave and all these great things that help us get ready for our day. But we aren't allowing our minds to deteriorate by using them, we are only making things easier for ourselves and saving time for other important things in our days. Going off to school or work in our cars instead of a horse and buggy. Think of the brainpower and genius that was used to come up with that single invention that has changed the way we move across this globe.Using technology to solve our continually more complicated problems as a human race is definitely a good thing. Our ability to think for ourselves isn't deteriorating, it's continuing to grow, moving on to higher though functions and more ingenious ideas. The ability to use what technology we have is an example.

  • Rater Commentary for Essay Response — Score 4
  • Developing a clear position, the author presents appropriate reasons to hold true and explain in support of the issue.
  • The point on technology usage is supported with relevant concepts that show how it enables users to access data and information and their potential that users would not normally have access to.
  • The ‘golden age’ point has also been appropriately supported by the basic description of our technologically saturated social condition.
  • Paragraph 3 on coffee pots to the benefits of technology to cars does showcase that the overall development and organization of this essay does live through the rare bouts of misdirection.
  • The essay seamless flows from one idea to another but often only scratching the surface level of the core issue rather than probing in-depth.
  • Taking the analysis further, the author creates a marked distinction between the essay and the level 3 response that supports the idea of technology advancing to help progress human thinking abilities, drawing an intelligent parallel between the promise of the modern sophisticated technology (computer) and the actual "impact" of equally "promising" and pervasive technologies of the past (printing press).
  • The reason why this essay has received a score of 4 as the language meets the expectations and the author has demonstrated a satisfactory control over it.
  • Overall, the mistakes are minor in nature and do not really hinder with the clarity of the notions being presented.

Essay Response Score 3

There is no current proof that advancing technology will deteriorate the ability of humans to think. On the contrary, advancements in technology had advanced our vast knowledge in many fields, opening opportunities for further understanding and achievement. For example, the problem of debilitating illnesses and diseases such as Alzheimer's disease is slowing being solved by the technological advancements in stem cell research. The future ability to grow new brain cells and the possibility to reverse the onset of Alzheimer's is now becoming a reality. This shows our initiative as humans to better our health demonstrates the greater ability of humans to think.

One aspect where the ability of humans may initially be seen as an example of deteriorating minds is the use of the internet and cell phones. In the past humans had to seek out information in many different environments and aspects of life. Now humans can sit in a chair and type anything into a computer and get an answer. Our reliance on this type of technology can be detrimental if not regulated and regularly substituted for other information sources such as human interactions and hands-on learning. I think if humans understand that we should not have such a reliance on computer technology, that we as a species will advance further by utilizing the opportunity of computer technology as well as the other sources of information outside of a computer. Supplementing our knowledge with internet access is surely a way for technology to solve problems while continually advancing the human race.

  • Rater Commentary for Essay Response — Score 3
  • The author has been able to transcend beyond the superficial strata of the core issue.
  • How technology has advanced overall human know-how in multiple domains and the way forward is to supplement the usage to “surely a way for technology to solve problems while continually advancing the human race."
  • However, the author has failed to offer sufficient evidence to support this point.
  • Point two is where the author has created an outline by citing the need for regulation/supplementation and cautions of the flipside of depending on technology heavily which is vague and restrictive - "Our reliance ... can be detrimental. If humans understand that we should not have such a reliance ... we will advance further."
  • Much of the second paragraph is filled with loosely connected generalizations which need more groundwork.
  • Some minor language errors are also present in this essay. However, the author’s meaning and implications are clear, thus earning this essay a score of 3.

Essay Response Score 2

In recent centuries, humans have developed the technology very rapidly, and you may accept some merit of it, and you may see a distortion in society that occurred by it. To be lazy for humans in some meaning is one of the fashion issues in these days. There are many symptoms and reasons for it. However, I can not agree with the statement that technology makes humans be reluctant to think thoroughly.

Of course, you can see the phenomena of human laziness along with developed technology in some places. However, they would happen in specific conditions, not general. What makes human to be laze of thinking is not merely technology, but the tendency of humans that they treat them as a magic stick and a black box. Not understanding the aims and theory of them courses the disapproval problems.

The most important thing to use thechnology, regardless of the new or old, is to comprehend the fundamental idea of them and to adapt suit tech to tasks in need. Even if you recognize a method as an all-mighty and it is extremely over-spec to your needs, you can not see the result you want. In this procedure, humans have to consider as long as possible to acquire adequate functions. Therefore, humans can not escape from using their brains.

In addition, the technology as it does not vain automatically is created by humans. Thus, the more developed tech and the more you want a convenient life, the more you think and emit your creativity to breakthrough some banal method sarcastically.

Consequently, if you are not passive to the new tech, but offensive to it, you would not lose your ability to think deeply. Furthermore, you may improve your ability by adopting it.

  • Rater Commentary for Essay Response — Score 2
  • This essay is no better than a score 2 simply because of the language used.
  • Serious errors in grammar, only specific moments of clarity, sentence mechanics and usage of terms minus meanings contribute to this.
  • If one were to overlook the obvious flaws, the author has made an effort to respond to the prompt - "I can not agree with the statement that the technology makes humans be reluctant to thinking thoroughly."
  • However, the author has not been successful in his assertion of - "Not understanding the aims and theory of them [technology] courses the disapproval problems" and "The most important thing to use the technology ... is to comprehend the fundamental idea of them."
  • Overall, the essay presents a badly inconsistent but not essentially lacking in an endeavor to produce and sustain its statements.

Essay Response Score 1

Humans have invented machines but they have forgotten it and have started everything technically so clearly their thinking process is deteriorating.

  • Rater Commentary for Essay Response — Score 1
  • The author’s manner of using significant terms from the prompt "technically" (technologically), "humans," "thinking" (think) and "deteriorating" (deteriorate) clearly demonstrates that the essay is topic and offers evidence of understanding.
  • It also shows the author’s inability to create an appropriate response that is in adherence to the instructions and guidelines provided - ("Discuss the extent to which you agree or disagree with the statement above and explain your reasoning for the position you take."
  • The language used also demonstrates that the essay is no better than a level 1 as the sentences formed are not coherent.

6 Tips for a Perfect-Scoring GRE ® Essay

Go over these fundamental pointers that should help you get that perfect score on your four GRE ® essays.

  • The common factor in all perfect-scoring GRE ® sample essays is an introduction and a conclusion.
  • Ideally, you need to introduce the core ideas in the first few lines, not necessarily a paragraph and ensure that you wrap them up at the end of the essay.
  • It is imperative that the author states their position on the issue clearly.
  • While you may scrape through a vague standpoint in an Argument essay and still score a 4.0, the same approach will not work in an Issue essay where you may just get a 3.0.
  • Regardless of the essay type or prompt, in order to get that perfect score, you will need to include a statement that clarifies your position and the extent to which you agree or disagree with it.
  • From the sample essays and their analysis, one aspect that is clearly identifiable is the need for the author to concentrate on relevant support for any claims or statements being made.
  • It is always more persuasive if the essay demonstrates claims that have been backed up by sufficient evidence than just making general points.
  • Therefore, going backward, if you are unable to find sufficient support for your arguments, then in all probability, that is a sign that you need to rethink your position or draw inspiration from another part of the argument that will withstand critique.
  • The communication of your notions and ideas and the support that you provide transcends polished writing, perfectly worded or spelt essays.
  • Issues that have been clearly-explained with compelling examples, evaluations and assessments by cutting through the heart of the argument is bound to get a higher score in comparison to imprecise essays that have been skillfully crafted.
  • A key factor in any of the essays getting a high score is a seamless and smooth transition of ideas between paragraphs that have been linked at multiple levels.
  • Even ideas within paragraphs should have linguistic transition levels.
  • All of the GRE ® essay samples adhere to the basic standard five paragraph routine that includes an introduction, body paragraphs and a conclusion.
  • The ETS website states, “You are free to organize and develop your response in any way you think will enable you to effectively communicate your ideas about the issue.”
  • But having your essay organized across these five paragraph formats it will save you the time of having to figure out a new organizational strategy for every essay you write.
  • And the more consistently you stick to a simple (but clear) organizational structure, the faster you’ll get at it until organizing your thoughts logically comes as second-nature.

Download your FREE eBook on AWA essay writing, now!

template for gre issue essay

1)(of food) enjoying the taste or smell for a long time 2)(of feeling or experience) enjoying the moment for a long time

Looking For

Top ms admits, get a call back from an admissions expert.

 alt=

Call me Back

Call me back.

  • Forgot Password?

New to GRE edge ? Sign Up here

Have a GRE edge account ? Login

About AdmitEDGE

AdmitEDGE (... formerly GREedge), Guidance Platform for Admissions & Tests provides one-on-one counseling and study programs that help students get better Admits to Universities, better Scores in GRE, TOEFL and better Bands in all aspects of English tested by IELTS.

  • Valued Epistemics Pvt Ltd. Unit:201, Floor 2, Indiqube Brigade Vantage, Kandanchavadi, Chennai - 600 096, India.
  • +91 98844 12858
  • [email protected]

AdmitEDGE - Our Products

Useful resources.

Recently viewed courses

Recently viewed.

Find Your Dream School

This site uses various technologies, as described in our Privacy Policy, for personalization, measuring website use/performance, and targeted advertising, which may include storing and sharing information about your site visit with third parties. By continuing to use this website you consent to our Privacy Policy and Terms of Use .

   COVID-19 Update: To help students through this crisis, The Princeton Review will continue our "Enroll with Confidence" refund policies. For full details, please click here.

How to Write a Great GRE Argument Essay

writing a GRE essay

When you take the GRE , you’ll have to write two essays : an Issue essay and an Argument essay. In your GRE Argument essay, you’ll get to demonstrate how well you can understand, analyze, and evaluate an argument. Here are ten GRE Argument essay tips you should know.

Fact #1: It doesn’t matter who is right

Fact #2: you'll have just 30 minutes for the gre argument essay, fact #3: graders will not pore over your essay, fact #4: quality matters, but so does quantity, fact #5: the prompt will tell you everything you need to know.

Make sure you read the prompt two or three times. You’ll want to make sure you truly understand it. Pay attention to what evidence is provided, what is stated in the prompt, and what is claimed by the author. A great way to identify fallacies is to determine what the author has assumed, and then try to explain why that assumption may be wrong. Here are four things to look for:

  • Lack of evidence to support an assumption : You’ll want to mention this dearth in your essay—and note the type of information that would strengthen the argument.
  • Non-specific language : Does the author make generalizations without providing specifics? You will want to point that out!
  • Jumping to conclusions : Most Argument prompts will jump to conclusions at least once. As you read each sentence in the prompt, look for the author’s reasoning. If you can’t find a clear line of argument, you should note that the author has jumped to conclusions.
  • Data values : Just because the author provides numbers doesn’t mean they’re necessarily objective or even true. Consider—and discuss within your essay—the reliability of any data, or data collection methods, that are presented in the prompt.

Fact #6: Structure will save you

After you read the prompt, brainstorm the logical fallacies you want to address. Then, choose your top three or four, and formulate a brief outline before you start your essay. There is nothing worse than having to stop writing your essay to come up with new ideas, so you’re going to want to follow a strict organizational format. Here’s a good general template to keep in mind:

  • Intro : This should consist of three or four sentences in which you provide an overview of all the fallacies you plan to address.
  • Fallacies : Each should get its own indented paragraph. You’ll want to discuss it in detail, and you may even opt to quote from the prompt in making your case.
  • Suggestions for improving the prompt argument : Time-permitting, you’ll ideally want to include a paragraph in which you detail how the author could make a stronger case.
  • Conclusion : As short as the introduction, this should summarize your body paragraphs (the fallacies and suggestions) and tie up any loose ends. Don’t skip this part! Even if you only have time for a single sentence, write one. An essay without a conclusion will almost certainly receive a lower score than one that is finished.

Fact #7: Clear writing is key

Fact #8: you’ll get one combined score for both essays, fact #9: you don’t have to be perfect to earn a perfect score, fact #10: you can plan ahead.

  • Graduate School  

Featured Grad Schools For You

Explore Graduate Programs for You

Explore our featured graduate schools & programs to find those that both match your interests and are looking for students like you.

Best Law Schools 2023

Best Law Schools

Check out our complete list of 168 law schools, based on surveys of school administrators and over 17,000 students.

Search for Medical Schools

Search for Medical Schools

Our medical school search allows you to refine your search with filters for location, tuition, concentrations and more.

Featured Business Schools For You

Find MBA Programs Matched to Your Interests

Explore our featured business schools to find those that are looking for students like you.

template for gre issue essay

Free MCAT Practice Test

I already know my score.

template for gre issue essay

MCAT Self-Paced 14-Day Free Trial

template for gre issue essay

Enrollment Advisor

1-800-2REVIEW (800-273-8439) ext. 1

1-877-LEARN-30

Mon-Fri 9AM-10PM ET

Sat-Sun 9AM-8PM ET

Student Support

1-800-2REVIEW (800-273-8439) ext. 2

Mon-Fri 9AM-9PM ET

Sat-Sun 8:30AM-5PM ET

Partnerships

  • Teach or Tutor for Us

College Readiness

International

Advertising

Affiliate/Other

  • Enrollment Terms & Conditions
  • Accessibility
  • Cigna Medical Transparency in Coverage

Register Book

Local Offices: Mon-Fri 9AM-6PM

  • SAT Subject Tests

Academic Subjects

  • Social Studies

Find the Right College

  • College Rankings
  • College Advice
  • Applying to College
  • Financial Aid

School & District Partnerships

  • Professional Development
  • Advice Articles
  • Private Tutoring
  • Mobile Apps
  • Local Offices
  • International Offices
  • Work for Us
  • Affiliate Program
  • Partner with Us
  • Advertise with Us
  • International Partnerships
  • Our Guarantees
  • Accessibility – Canada

Privacy Policy | CA Privacy Notice | Do Not Sell or Share My Personal Information | Your Opt-Out Rights | Terms of Use | Site Map

©2024 TPR Education IP Holdings, LLC. All Rights Reserved. The Princeton Review is not affiliated with Princeton University

TPR Education, LLC (doing business as “The Princeton Review”) is controlled by Primavera Holdings Limited, a firm owned by Chinese nationals with a principal place of business in Hong Kong, China.

Get GRE Prepared with Yocket Prep

Take a diagnostic test, set up your personalized dashboard, and find a pool of practice tests. Explore our packages to learn more!

Table of Content

  • GRE AWA Template Format

How to Write GRE AWA Essays?

Prepare for gre with yocket prep, frequently asked questions about gre awa template, gre awa template: how to write the gre awa essays.

gre-awa-template

The GRE analytical writing assessment tests your analytical and writing skills. The assessment has a time duration of 60 minutes, and entails 2 tasks. The first being “Analyse an Issue” and the other being  “Analyse an Argument”. As the name itself suggests, the "Analyse an Issue” task assesses your understanding of an issue and how well you interpret it with reasons. The other section analyses given arguments through logical reasons.

Your essay must be convincing and easy to comprehend if you wish to score good in this section. Whether you are confident about the GRE AWA section or not, practice is the key to help you complete this section within the given time frame. Here we have prepared a complete guide with GRE analytical writing template, suggestions for writing better while also managing your time and also help you understand how your skills will be measured and scored by raters.

GRE AWA Template Format 

First of all, we need to understand the essay format using AWA template GRE . The task is to time yourself properly so as to read, comprehend, compose and answer within 30 minutes. The basic GRE analytical writing template must comprise 5-paragraphs while some of you may prefer to have 4 or 6 paragraphs depending on how you divide the body of the essay Here is what the basic 5-paragraph GRE AWA template must include:

  • Introduction
  • Body (Para 1)
  • Body (Para 2)
  • Final Body (Para 3)

Let us closely discuss what each of these paragraphs must highlight in your GRE AWA template:

1. Introduction

To start the essay, you must first state the given issue or topic and briefly explain it in your own words. This part is the kickstart of your essay and is very important to catch the attention of the reader. In the final few lines of your introduction, introduce what’s coming next for the reader to hold their attention. Try to keep the introduction as specific as possible such that it shows you have clearly understood the central idea of the topic.

2. Body (Para 1)

This is the part where you must put your best foot forward i.e. come up with the best and most relevant example or argument you have in mind. You can bring up examples from any background, be it social, political, cultural, historical, and on and so forth, but you must be confident that it matches your position in the essay.

3. Body (Para 2)

Being the 3rd paragraph of your essay, you must start it with a smooth transition from the previous paragraph and follow the same pattern of giving an example along with a suitable explanation and reinstating your thesis in the essay. If you wish to keep 4 paragraphs in your essay, this will be the final body paragraph and you will move on to the conclusion.

4. Final Body (Para 3)

Decide a transition phrase again before citing another example. Since this is the 3rd example, keep in mind that it is something other than what you have already specified in the above paragraphs. Wind up your discussion by gradually pointing towards the conclusion.

5. Conclusion

The last part of the GRE AWA essay is the conclusion which is equally important as the other parts in the paragraph. So don’t be in haste to complete the essay by mentioning general lines at the end to fulfil the word limit. This is where you summarise the entire essay in a few lines and also refute the opposite viewpoint. Include a few analytical statements to make the conclusion effective.

prep-banner

Now that you have learnt about the template for GRE issue essay and argument essay, let us take a look at some useful tips. Here are a few suggestions that will help you to structure GRE argument and GRE issue essay template better:

  • Practice is the key to perfection, therefore rather than being worried about your paper, keep practising various topics that are relevant to the real world.
  • While framing your essay try to be concise and relevant.
  • A well-organised essay following GRE AWA templates is always preferred.
  • While citing examples, see that your example does not occupy most part of the essay.
  • The central idea and your take on it must be emphasised upon.
  • Writing in first person speech is not a very good idea especially for the introduction and conclusion.
  • A smart way to conclude your essay is by mentioning the opposing views and then invalidating it by stating your own reasoning and ending it with a strong conclusive statement.
  • At the end, try to save yourself a few minutes to give a quick read to your essay and spot any grammatical errors so that it does not affect your scores.

This was all about the GRE AWA essay template. To ace the GRE AWA, you must look at sample topics, previous year responses, the GRE AWA template, marking scheme and understand expected topics. There is no restriction on the subject area from which questions can be expected. It could be related to social sciences, humanities, fine arts, etc. so keep exploring topics from all these subjects.

Suggested : GRE General Test vs Subject Test

Whatever mode you decide to take the GRE test, we will be there for you. 

With Yocket’s GRE Prep platform you can get hold of a whole lot of things - FREE mock & diagnostic tests , FREE resources , more than 40 personalised difficulty-level subject practice tests with instant performance report and detailed solutions, and much much more! 

And well, not just that, Yocket Prep will offer you: 

  • Instant solutions and performance analysis based on your diagnostic tests
  • Focussed and prioritised GRE learning and practice schedule, to keep you ahead of time 
  • Custom priority-based study planner, which focuses on the most important & marks-fetching topics practice by volume
  • 7 GRE style tests (which are very difficult to get tbh), 40+ topic-wise tests, access to Priority Dashboard and much more than you can imagine through Yocket Prep+

For better guidance, join Yocket premium today and get specially curated suggestions, counselling services, documentation support and get answers to your most commonly asked questions about entrance tests and studying abroad from experienced counsellors! 

Continue Reading

Articles you might like

blog_image

GRE AWA Sample Essays: Analytical Writing Examples

Apr 11, 2024 • 6 mins read

blog_image

GRE AWA Preparation: How to Prepare for the GRE AWA Section

Apr 19, 2024 • 8 mins read

blog_image

How to Crack GRE: Tips & Strategies on How to Crack GRE Exam?

Jan 8, 2024 • 4 mins read

How should I start my GRE AWA issue?

What are the main criteria for evaluation of gre awa, how long should the gre awa essay be, are gre awa scores important for admission to ms, how can i score 4.0 or above in gre awa, a test-taking platform that helps for gre® preparation online., other links.

  • Help Center
  • Privacy Policy

OUR PRODUCTS

© 2024 Yocket Prep. All rights reserved.

GRE® is a registered trademark of Educational Testing Service (ETS). This website is not endorsed or approved by ETS.

Notice about upcoming GregMat price increase Notice about upcoming GregMat price increase

  • Executive Assessment
  • Bocconi Test
  • Testimonials

GRE Argument Essay Template and Sample Argument Essay

  • April 6, 2023

gre argument essay

The second essay you’ll have to write when you sit down to take the GRE is the GRE Argument Essay. It’s only 30 minutes and can be a bit of a challenge…

So you need a strong game plan.

But should you really worry about the essays since they don’t factor into your quantitative and verbal scores?

Absolutely!

We tell our students that while the essay scores may not be of importance to graduate programs, the ease of writing them should be important to you. The Analysis of an Issue and Argument essays are the first sections of the GRE. You don’t want to go into them frazzled and anxious. You definitely don’t want to finish them feeling like you could have done better. Unfortunately, this anxiety will carry into the rest of the test and NOT pave the road to earning your best GRE score.

To help you relax into the essays and the rest of the test, we’re sharing the exact template and sample essay we show our GRE students. Before you take a look, read these tips on how to practice with them.

GRE Argument Essay Practice Tips

gre issue essay template and sample

1. Words to use in every argument essay are in blue on the template.

You want to have these transitions and links well practice so that you can use them automatically on the test. Obviously, this essay is HIGHLY dependent on the one provided in the prompt, so much of what you write is unique to the question.

2. Your GRE argument essay should be 4 or 5 paragraphs depending on the question.

Some questions ask you to evaluate another opinion, discuss a disadvantage, or give an opposing viewpoint. Therefore, you need another body paragraph to respond to that part of the prompt.

3. The sample GRE argument essay is directly from ETS, so don’t copy the essay.

Think about how your examples and reasons can be used to respond to the prompt.

PEEL means Point – Example – Explanation – Link. The blue in the sample essay shows how well the body paragraphs conform to the PEEL writing format.

5. Always outline your essay first.

It is very obvious to a trained reader when students don’t outline their essays first. You just have to jot down your opinion and reasons or examples in a quick list. This quick outline will help keep you focused and make your writing more cohesive.

6. Check out the full list of GRE argument essay prompts .

You shouldn’t write essays for each prompt (there are a lot). However, you should read over the whole list looking for differences in the ways questions are asked. Then, pick 10 prompts to practice outlining. Of those 10, pick 5 essays to write fully.

7. For extra practice, work on the parts, rather than the whole.

Just write an introduction, a body paragraph, or a conclusion for some of the essay prompts. After outlining, of course. This type of focused writing will improve the individual parts of the essay, resulting in an entire essay that is much better overall. You don’t have to write the parts for the same essay. Switch it up!

8. When you write a full essay, do it in 30 minutes.

Practice the way you are going to perform. By the time you write your second full essay, you should be writing in only 30 minutes. Set a timer and type the essay on the computer without using spell check or Grammarly. Ideally, you will write the essays when you do a full-length GRE practice test . It’s important to develop stamina for the full 4-hour test.

GRE Argument Essay Template

In the argument provided [or letter or memorandum] , the author asserts that [his recommendation or proposal]. The author’s argument does not make a persuasive case for [goal of the argument]. This argument lacks sufficient logical reasoning and appropriate evidence, and thus, is not strong enough to [goal of the argument].

First, write your first reason that the argument is not sound here. For example, put your example of the assumptions the argument depends on here. You can use quotation marks if you directly reference the text. Explain your example with consideration of the stated and unstated assumptions. To strengthen her argument, the author would benefit from add the link to your thesis about the argument here.

In addition, write your second reason that the argument is not sound here. For instance, put your example here. Explain your example with consideration of the stated and unstated assumptions. Unless add specific link to the argument, it can not be used to effectively back the author’s argument.

[You may need another paragraph depending on what the prompt asks you to do.]

The conclusion should consist of a restatement of the argument goal, a restatement of your thesis, and a closer. The location of these in the paragraph will be different based on how you choose to write it.  

GRE Argument Essay Sample Prompt

In surveys, Mason City residents rank water sports (swimming, boating and fishing) among their favorite recreational activities. The Mason River flowing through the city is rarely used for these pursuits, however, and the city park department devotes little of its budget to maintaining riverside recreational facilities. For years there have been complaints from residents about the quality of the river’s water and the river’s smell. In response, the state has recently announced plans to clean up Mason River. Use of the river for water sports is therefore sure to increase. The city government should for that reason devote more money in this year’s budget to riverside recreational facilities.

Write a response in which you examine the stated and/or unstated assumptions of the argument. Be sure to explain how the argument depends on the assumptions and what the implications are if the assumptions prove unwarranted.

Sample GRE Argument Essay

[Restatement of the given argument] In the argument provided, the author asserts that the Mason City government ought to devote more money to riverside recreational facilities. [Your thesis] The author’s argument does not make a cogent case for increased resources based on river use. [Overview of reasons related to goal] This argument is rife with holes and assumptions, and thus, not strong enough to lead to increased funding.

[First reason] Citing surveys of city residents, the author reports city resident’s love of water sports. It is not clear, however, the scope and validity of that survey. [Example] For example, the survey could have asked residents if they prefer using the river for water sports or would like to see a hydroelectric dam built, which may have swayed residents toward river sports. [Explanation] The sample may not have been representative of city residents, asking only those residents who live upon the river. The survey may have been 10 pages long, with 2 questions dedicated to river sports. We just do not know. [Link] Unless the survey is fully representative, valid, and reliable, it can not be used to effectively back the author’s argument.

[Second reason] Additionally, the author implies that residents do not use the river for swimming, boating, and fishing, despite their professed interest, because the water is polluted and smelly. [Explanation] While a polluted, smelly river would likely cut down on river sports, a concrete connection between the resident’s lack of river use and the river’s current state is not effectively made. Though there have been complaints, we do not know if there have been numerous complaints from a wide range of people, or perhaps from one or two individuals who made numerous complaints. [Link] To strengthen his/her argument, the author would benefit from implementing a normed survey asking a wide range of residents why they do not currently use the river.

[Third reason] Building upon the implication that residents do not use the river due to the quality of the river’s water and the smell, the author suggests that a river clean up will result in increased river usage. If the river’s water quality and smell result from problems which can be cleaned, this may be true. [Example] For example, if the decreased water quality and aroma is caused by pollution by factories along the river, this conceivably could be remedied. But if the quality and aroma results from the natural mineral deposits in the water or surrounding rock, this may not be true. [Explanation] There are some bodies of water which emit a strong smell of sulphur due to the geography of the area. This is not something likely to be afffected by a clean-up. Consequently, a river clean up may have no impact upon river usage. [Link] Regardless of whether the river’s quality is able to be improved or not, the author does not effectively show a connection between water quality and river usage.

[Closer] A clean, beautiful, safe river often adds to a city’s property values, leads to increased tourism and revenue from those who come to take advantage of the river, and a better overall quality of life for residents. [Restatement of the given argument/goal] For these reasons, city government may decide to invest in improving riverside recreational facilities. [Restatement of thesis] However, this author’s argument is not likely significantly persuade the city goverment to allocate increased funding.

Need help preparing for the GRE?

Happy students are our business.

graduate testimonial panel

More from Apply Me

dubai gre tutors worth it

Leave a Reply Cancel reply

Your email address will not be published. Required fields are marked *

Save my name, email, and website in this browser for the next time I comment.

WhatsApp us

  • Share full article

Advertisement

Supported by

news analysis

Necessity Gives Rise to Bipartisanship — for Now

The far right finds itself marginalized in the House as Speaker Johnson pushes through aid to Ukraine and Israel by relying on Democrats.

Speaker Mike Johnson looks to the side while surrounded by a crowd in the Capitol. He is wearing a dark suit with a maroon tie.

By Carl Hulse

Reporting from Capitol Hill

When Congress convened in 2023, an empowered far-right Republican faction in the House threatened to upend Washington and President Biden’s agenda.

But the intransigence of that bloc instead forced Republicans and Democrats into an ad hoc coalition government that is now on the verge of delivering long-delayed foreign military aid and a victory to the Democratic president.

The House approval on Saturday of money for Ukraine, Israel and Taiwan over angry objections from the extreme right was the latest and perhaps most striking example of a bipartisan approach forged out of necessity. The coalition first sprang up last year to spare the government a catastrophic debt default, and has reassembled at key moments since then to keep federal agencies funded.

Unable to deliver legislation on their own because of a razor-thin majority and the refusal of those on the right to give ground, House Republicans had no choice but to break with their fringe members and join with Democrats if they wanted to accomplish much of anything, including bolstering Ukraine in its war against Russia.

“Look at what MAGA extremism has got you: nothing,” Representative James P. McGovern, Democrat of Massachusetts, told Republicans on the House floor as lawmakers took their first steps toward approving the aid package. “Nothing. Not a damn thing. In fact, it has empowered Democrats. At every critical juncture in this Congress, it has been Democrats who have been the ones to stand up for our country and do the right thing for the American people.”

The moments of bipartisan coming-together are hardly a template for a new paradigm of governing in polarized times. The grudging G.O.P. collaboration with Democrats has only come about on truly existential, must-pass legislation — and typically only at the last minute after Republicans have exhausted all other options, making the coalition unlikely to hold on less critical bills and the social policy issues that sharply divide the two parties.

And the political incentives are stacked decisively against it. The cooperation with Democrats has placed Speaker Mike Johnson at risk of losing his post, making him the second G.O.P. speaker to face a threat to his job for reaching across the aisle, after Kevin McCarthy was toppled last year.

With its legislative power diluted, the furious right has been left to wield the motion to vacate the speaker’s chair as its only remaining weapon.

“This is a sellout of America,” Representative Marjorie Taylor Greene, the Georgia Republican who has taken steps to try to force Mr. Johnson from the speakership, said after the vote.

The few instances of coalition governing also have come about grindingly slowly. Mr. Johnson delayed for months as he deliberated over whether to move forward with the Ukraine element of the legislation and put his speakership on the line. It had been clear for months that the aid would pass overwhelmingly if it only it was put on the floor, and the lopsided vote totals on Saturday were probably not substantially different than they would have been if the vote had been held many months ago.

“I call it failing through the day to a good conclusion,” said Representative Patrick T. McHenry, the North Carolina Republican who temporarily served as speaker after Mr. McCarthy was deposed. “The frustration here is that we are going through the worst set of policymaking and taking an excruciatingly long period of time to go through what is an inevitable result. It is long past frustrating.”

Mr. McHenry was not the only one feeling that way. As they have watched their priorities and plans get steamrolled by the bipartisan coalition, those on the far right have grown increasingly exasperated as members of their own party align with Democrats to override their strident opposition.

“There is continued frustration with the fact that we are, frankly, allowing the House to be governed by Democrats,” said Representative Chip Roy, Republican of Texas. “Every single point of leverage has been given away in abject failure and capitulation from Day 1.”

While Democrats say the foreign aid package should have been approved months ago, they took some satisfaction in seeing the marginalization of the far right.

“They should have been made irrelevant a long time ago,” said Representative Steny H. Hoyer, Democrat of Maryland and a former House majority leader. “The problem was we sent a message for two or three months of indecisiveness in America. Indecisiveness, and a lack of resolve to confront an invader, an autocratic invader of a free country. And we also sent a message of lack of resolve on Israel confronting terrorists.”

Democrats have not gotten all they wanted in their often difficult and halting negotiations with the Republicans that at times threatened the financial stability of the federal government.

Mr. Biden had to agree to spending caps to avert a federal default that would have been caused by breaching the debt limit last year, setting off a spending fight that was not resolved until March. Democrats also had to swallow some spending cuts to favored programs such as I.R.S. enforcement. But in many respects, the spending parameters for the year — and in the military aid package — were shaped by Democrats, as evidenced by the strong support from the party in the end.

“I am glad to see the House finally moving forward to pass this critical legislation, which mirrors the package I negotiated and helped pass here in the Senate,” said Senator Patty Murray, the Washington Democrat who chairs the Appropriations Committee.

When it came to the money to sustain Ukraine, Democrats also had the advantage of strong support in Senator Mitch McConnell, the Kentucky Republican and minority leader, who was unyielding in his backing of the financial assistance despite dwindling support for it among his fellow Senate Republicans.

Mr. McConnell’s stance ensured a sufficient number of Senate Republicans would be on board. It also meant three of the four congressional leaders — himself; Senator Chuck Schumer, the New York Democrat and majority leader; and Representative Hakeem Jeffries of New York, the House Democratic leader — were all strongly behind the aid to Ukraine along with Mr. Biden, putting immense pressure on Mr. Johnson to join them.

The intense effort to deliver the Ukraine aid also exposed the limits of the coalition approach. With Republicans demanding new border security provisions as part of any ultimate agreement, a bipartisan bloc of senators engaged in prolonged talks that in February produced a proposal that included significant Democratic concessions aimed at stopping the flow across the border. But the plan was immediately torpedoed by former President Donald J. Trump and other Republicans unwilling to let go of a powerful campaign issue.

With the fight over the Ukraine funding drawing to a close, Congress has just a handful of legislative issues it must deal with this year — a Pentagon policy measure, a farm bill, renewal of Federal Aviation Administration programs and most likely a temporary measure to fund the government through November. Given divided control of government, all that legislation will need to be advanced on a bipartisan basis.

But the steady approach of elections that will decide control of both chambers of Congress and the White House means much of the time will be taken up by the parties lobbing political grenades at one another, meaning bipartisanship could be difficult to come by in the months ahead.

Carl Hulse is the chief Washington correspondent, primarily writing about Congress and national political races and issues. He has nearly four decades of experience reporting in the nation’s capital. More about Carl Hulse

A Divided Congress: Latest News and Analysis

House Approves $95 Billion Aid: The House voted resoundingly to  approve $95 billion  in foreign aid for Ukraine, Israel and Taiwan, as Speaker Mike Johnson put his job  on the line to advance  the long-stalled aid package by marshaling bipartisan support. Here’s the breakdown  and how the House voted .

Extension of Surveillance Law: The Senate approved an extension of a warrantless surveillance law, sending President Biden legislation that national security officials say is crucial to fighting terrorism but that privacy advocates decry  as a threat to Americans’ rights.

A Divided G.O.P.: The House vote on Ukraine aid was the clearest sign yet that at least on foreign policy, the Republican Party is not fully aligned  with former President Donald J. Trump and his “America First” movement .

TikTok Bill: The House made another push to force through legislation that would require the sale of TikTok by its Chinese owner or ban the app in the United States by packaging the measure with aid to Ukraine and Israel .

Mayorkas Impeachment: Republicans say the Senate’s quick dismissa l of charges against Alejandro Mayorkas, the homeland security secretary, sets a dangerous precedent. Democrats say the mistake would have been to treat the case seriously .

Campus Antisemitism Hearing: Columbia’s president, Nemat Shafik, agreed that the university needed to take a tougher stance on antisemitism, in response to harsh questioning from a Republican-led House committee .

IMAGES

  1. Find & Download GRE Analytical Writing Templates Online

    template for gre issue essay

  2. How to Write the Revised GRE Analytical Writing Essays

    template for gre issue essay

  3. 011 How To Write Gre Essay Example Prompts Goal Blockety Co Analytical

    template for gre issue essay

  4. Writing About Enduring Issues: Essay Which Matters

    template for gre issue essay

  5. Writing the gre argument essay step by step guide

    template for gre issue essay

  6. Gre Essay Template

    template for gre issue essay

VIDEO

  1. Best notion content planner template

  2. [TEMPLATE] Solo Render Prisma3D

  3. Premiere Pro Letter Box Tutorial

  4. Vn video Editing

  5. Import Office 2016 template into Group Policy

  6. Issue Essay

COMMENTS

  1. How to Structure the GRE Issue Essay

    The GRE Issue essay is similar in structure to the classic 5-paragraph short essay. You may opt for 4-6 paragraphs, but the template given here plans for 5. The official GRE website states that readers of the Issue essay "are evaluating the skill with which you address the specific instructions and articulate and develop an argument to ...

  2. GRE General Test Analytical Writing Analyze an Issue Task

    The "Analyze an Issue" task assesses your ability to think critically about a topic of general interest and to clearly express your thoughts about it in writing. Each Issue topic makes a claim that can be discussed from various perspectives and applied to many different situations or conditions. Your task is to present a compelling case for ...

  3. GRE Issue Essay Tips and a Bulletproof Template

    GRE Issue Essay Example That Received A Perfect Score You may notice that many Issue Essay prompts make statements that are difficult to fully support. The example and response I want to use first is this one about technology (link is to the full sample essay on the GRE's website). Go ahead and read the prompt, then read the "6" response just ...

  4. 4 Top-Scoring GRE Sample Essays, Analyzed (Issue

    Issue Essay 1: Technology and Human Ingenuity. The first of the GRE sample essays we'll be looking at is written in response to the following "Analyze an Issue" prompt: As people rely more and more on technology to solve problems, the ability of humans to think for themselves will surely deteriorate.

  5. PDF The GRE Analytical Writing Templates

    Fill in the Essay Template This exercise is going to be a hands-on experience for you. We have given the templates below for both argument and issue essays, and there is a text box at the end of each instruction. To practice writing AWA essays, all you need to do is: 1. Pick a random AWA question from the pool of essays.

  6. PDF Analytical Writing Sample Essays and Commentaries Large Print (18 point

    The Analytical Writing portion of the GRE®General Test consists of two writing topics, an Issue topic and an Argument topic. This document contains the writing topics for Practice Test #3, the scoring guides for each section, and sample responses with commentaries for each topic. Note: Sample responses are reproduced exactly as written ...

  7. The New GRE AWA: Tips to Approach the GRE Essay

    The new GRE Analytical Writing Assessment (AWA) consists of the Issue essay. You are allotted 30 minutes for your essay. You are allotted 30 minutes for your essay. The GRE essay tests your ability to write a cogent thesis statement that you must defend over the course of several paragraphs.

  8. GRE Issue Essay: Strategies + 8 Real Student Essays with Scores

    Issue Essay Analysis. Score: 5.5. This GRE Issue essay starts off with a strong intro that clearly articulates the author's position. The essay is also very long, and the body paragraphs well developed. In terms of ideas this is a strong—though if slightly limited—essay. It makes a compelling case for interdisciplinary learning.

  9. FREE 300-page GRE Course: Analytical Writing Essays (sample templates

    Introductory Paragraph (2-4 sentences) Try to accomplish three goals in your introductory paragraph: Briefly restate the argument in your own words. Briefly trace the argument's line of reasoning. Indicate the extent to which the argument is logically convincing. If possible, sum up your arguments in one sentence (or two brief sentences).

  10. GRE Issue Essay: 4 Steps to a Perfect Score • PrepScholar GRE

    4 Steps to a Perfect GRE Issue Essay. As a summing-up of all the information in this article, I'll go over the four essential GRE Issue essay tips to reliably achieve a high score. #1: Include a Clear Thesis. To fulfill the basic requirements of any GRE Issue essay task, you need to make your position on the issue clear.

  11. PDF Sample GRE ® Issue Task with Strategies, Sample Essay Responses and

    Here is a sample Issue task that you might encounter on the GRE Analytical Writing measure: As people rely more and more on technology to solve problems, the ability of humans to think for themselves will surely deteriorate. Discuss the extent to which you agree or disagree with the statement and explain your reasoning for the position you take ...

  12. GRE Issue Essay: Elements, Structure, Strategies & Samples

    Suggested: GRE AWA Sample Essays. 5 Strategies to Ace the GRE Issue Essay. 1. Structure is key. Even a passionate and well-reasoned response can fall flat without a solid essay structure. The classic format with an introduction, body paragraphs, and a conclusion ensures your message is clear and organized. 2.

  13. GRE Issue Essay: Practice Sample Prompts

    GRE Issue Essay Sample Prompt #1. 1. The emergence of the online "blogosphere" and social media has significantly weakened the quality of political discourse in the United States. Reason: When anyone can publish political opinions easily, standards for covering news and political topics will inevitably decline.

  14. GRE Essay Prompts

    The GRE Analytical Writing section requires you to write two essays—one will be an analysis of an issue and the other will be an analysis of an argument. You will have 30 minutes for each essay. Try your hand at these GRE essay prompts, and read our explanations for what makes a great GRE essay. We pulled these sample questions from our book ...

  15. GRE General Test Analytical Writing Analyze an Argument Task

    To get a clearer idea of how GRE raters apply the Argument scoring criteria to actual essays, you should review scored sample Argument essay responses and rater commentary. The sample responses, particularly those at 5 and 6 score levels, will show you a variety of successful strategies for organizing and developing an insightful evaluation.

  16. GRE Issue Essay Template and Sample Issue Essay

    The sample GRE issue essay is directly from ETS, so don't copy the essay. Think about how your examples and reasons can be used to respond to the prompt. 4. PEEL! PEEL means Point - Example - Explanation - Link. The blue in the sample essay shows how well the body paragraphs conform to the PEEL writing format. 5.

  17. GRE Analytical Writing Templates

    Typically, both essays should use the "Introduction-Body-Body-Body-conclusion" structure. Essays that are awarded perfect scores usually have five paragraphs with substantial examples fitted into the body paragraphs. An effective issue essay will present an insightful take of the given topic, along with compelling reasons or persuasive ...

  18. How to Write a Perfect GRE Issue Essay: Tips, Tricks, & Examples

    Paragraph 2: Main Argument - Give evidence as to why you believe that point of view. Paragraph 3: Counterargument - Address the other side of the argument, but quickly refute it by giving evidence to prove your original side of the argument. Paragraph 4: Conclusion - Conclude your essay. Do not merely summarize.

  19. Nail the GRE Analytical Writing with Sample Issue Essays

    The analytical writing section is the first part of the GRE and is designed to assess your critical thinking and analytical writing skills. It consists of two 30-minute tasks: the 'Analyze an Issue' task and the 'Analyze an Argument' task. In the issue task, you are given a statement on a general issue, and you are required to write a ...

  20. GRE Analytical Writing Sample Essays

    The GRE ® Issue essay is similar in structure to the classic 5-paragraph short essay. You can go through the following links for familiarizing yourself with GRE ® sample essays pertaining to the Issue task. GRE ® Issue Essay-1: "We learn through direct experience; to accept a theory without experiencing it is to learn nothing at all."

  21. How to Write a Great GRE Argument Essay

    Fact #4: Quality matters, but so does quantity. The essays that tend to get the highest grades have one feature in common: length! Write as much as you can—without being repetitive. Your Argument essay should include at least four indented paragraphs and consist of 350-600 words—ideally somewhere in the 500-600-word range.

  22. GRE AWA Template: How to Write the GRE AWA Essays?

    Body (Para 1) Body (Para 2) Final Body (Para 3) Conclusion. Let us closely discuss what each of these paragraphs must highlight in your GRE AWA template: 1. Introduction. To start the essay, you must first state the given issue or topic and briefly explain it in your own words. This part is the kickstart of your essay and is very important to ...

  23. Writing the GRE Issue Essay

    Greg Mat+ users, you can find the Greg Mat flashcards app on the App Store or the Google Play Store. Greg Mat+ users, you can find the Greg Mat flashcards app on the App Store or the Google Play Store.

  24. GRE Argument Essay Template and Sample Argument Essay

    The sample GRE argument essay is directly from ETS, so don't copy the essay. Think about how your examples and reasons can be used to respond to the prompt. 4. PEEL! PEEL means Point - Example - Explanation - Link. The blue in the sample essay shows how well the body paragraphs conform to the PEEL writing format. 5.

  25. House Aid Package Is Latest Example of Bipartisanship Forged Out of

    The House approval on Saturday of money for Ukraine, Israel and Taiwan over angry objections from the extreme right was the latest and perhaps most striking example of a bipartisan approach forged ...